SlideShare a Scribd company logo
1 of 103
Arterial Blood Gas
  Interpretation

  PRESENTER- Dr. Shankerdeep Sondhi
                    Resident IIInd Yr
             Department of Medicine
OBJECTIVES
ABG    Sampling

Interpretation   of ABG

Case   Scenarios
ABG – Procedure and Precautions
   Site- (Ideally) Radial Artery
                   Brachial Artery
                   Femoral Artery

   Ideally - Pre-heparinised ABG syringes
            - Syringe should be FLUSHED with 0.5ml
        of 1:1000 Heparin solution and emptied.
        DO NOT LEAVE EXCESSIVE HEPARIN IN THE
                              SYRINGE

HEPARIN             DILUTIONAL                   HCO3
                       EFFECT                     PCO2
    Only small 0.5ml Heparin for flushing and discard it
    Syringes must have > 50% blood.
   Ensure No Air Bubbles. Syringe must be sealed immediately
    after withdrawing sample.
    ◦ Contact with AIR BUBBLES
    Air bubble = PO2 150 mm Hg , PCO2 0 mm Hg
    Air Bubble + Blood = PO2       PCO2

   ABG Syringe must be transported at the earliest to the
    laboratory for EARLY analysis via COLD CHAIN
ABG ELECTRODES
A. pH (Sanz Electrode)
 Measures H+ ion concentration of sample against a
 known pH in a reference electrode, hence potential
 difference. Calibration with solutions of known pH (6.384
 to 7.384)

B. P CO2 (Severinghaus Electrode)
 CO2 reacts with solution to produce H+
   higher C02- more H+  higher P CO2 measured

C. P 02 (Clark Electrode)
 02 diffuses across membrane producing an electrical
 current measured as P 02.
Interpretation of ABG
Acid Base Homeostasis
1.   Plasma Acid Homeomostasis: Chemical
     buffering

     ◦ H+ influenced by
       Rate of endogenous production
       Rate of excretion
       Buffering capacity of body

     ◦ Buffers effective at physiologic pH are
         Hemoglobin
         Phosphate
         Proteins
         Bicarbonate
2. Alveolar Ventilation:
  pCO2 action is immediate. Stimulation of respiratory
  center washes off the excess CO2 increasing pH, and
     vice versa
3. Excretion:
  Liver: uses HCO3– to make urea which further
     prevents accumulation of ammonia and traps H+ in
     renal distal tubule
  Kidney: regulate plasma [HCO3–] through three main
     processes:
  • reabsorption of filtered HCO3–
  • formation of titratable acid
  • excretion of NH4+ in the urine.
  Proximal tubule reclaims 85% filtered HCO3–
  Distal tubule reclaims 15%, and excretes H+
  40–60 mmol/d of protons is excreted to maintain balance
Acid Base Balance
 H+ ion concentration in the body is
  precisely regulated
 The body understands the importance of H+
  and hence devised DEFENCES against any
  change in its concentration-

BICARBONATE    RESPIRATORY      RENAL
BUFFER         REGULATION       REGULATION
SYSTEM         Acts in few      Acts in hours to
Acts in few    minutes          days
seconds
BUFFER SYSTEM
 1st line of defence in pH regulation.
 Determine capacity of ECF to
  transport acids from site of production
  to site of excretion without undue
  change in pH.
 They resist change in pH on addition
  of acid/alkali in media wherever they
  are.
PLASMA
                RBC BUFFERS
BUFFERS


 NaHCO3/H2CO    KHCO3/H2CO3
  3              K2HPO4/KH2P4
 Na2HPO4/       KHb/HHb
  NaH2PO4
 Na-Pr/H-Pr
BICARBONATE BUFFER
   NAHCO3/H2CO3 = 20/1 = Alkali reserve.
   conversion of strong & non volatile acid
    in ECF, in weak & volatile acid at the
    expense of NaHCO3 component of
    buffer.
   H2CO3 thus formed is eliminated by lung
    as CO2.Directly linked with respiration &
    healthy lungs essential for its function.
   High concentration in blood,so very good
    physiological buffer.
   Weak chemical buffer.
 Bicarbonate   Buffer System

CO2 + H2O carbonic anhydrase H2CO3    H+ + HCO3-

     In Acidosis - Acid = H+
  H+ + HCO3       H2CO3      CO2 + H2O

   In Alkalosis - Alkali + Weak Acid = H2CO3
  CO2 + H20      H2CO3       HCO3- + H+
                    +
                ALKALI
PHOSPHATE BUFFER
 Na2HPO4/NaH2PO4 =
  AlkPO4/AcidPO4 = 4/1.
 NaH2PO4 excreted by kidneys.
  Directly linked with kidneys & healthy
  kidneys necessary for its functioning.
 Concentration in blood is low so not
  good physiological buffer.
 Very good chemical buffer as pKa
  aprroches pH.
PROTEIN BUFFER
 Na+Pr- / H+Pr- = salt/acid.
 Proteins can act as a base In acidic
  medium nd as an acid in basic
  medium..with COOH& NH2 group.
 Buffering capacity of plasma proteins
  is much less than Hb.
 Respiratory Regulation   of Acid Base
 Balance-


                   ALVEOLAR
  H+               VENTILATION       PaCO2




                    ALVEOLAR
  H+                VENTILATION      PaCO2
 Renal   Regulation of Acid Base Balance
Kidneys control the acid-base balance by excreting
 either an acidic or a basic urine,
This is achieved in the following ways-


Reabsorption                                      Secretion of H+
 of HCO3                                           ions in tubules
 in blood                                          and excretion
             PCO2                                 K+
             in ECF
                                                       Aldosterone
    H+ ion            •Proximal Convulated
                      Tubules (85%)
                      •Thick Ascending Limb of
ECF Volume            Loop of Henle (10%)
                                                        Angiotensin II
                      •Distal Convulated Tubule
                      •Collecting Tubules(5%)
Definitions and Terminology

 ACIDOSIS   – presence of a process which tends to
    pH by virtue of gain of H + or loss of HCO3-
 ALKALOSIS – presence of a process which tends
  to pH by virtue of loss of H+ or gain of HCO3-

If these changes, change pH, suffix ‘emia’ is added
 ACIDEMIA – reduction in arterial pH (pH<7.35)
 ALKALEMIA – increase in arterial pH (pH>7.45)
 Simple Acid  Base Disorder/ Primary Acid Base
 disorder – a single primary process of acidosis or
 alkalosis due to an initial change in PCO2 and HCO3.

 Compensation   - The normal response of
 the respiratory system or kidneys to change in pH
 induced by a primary acid-base disorder
 The Compensatory responses to a primary Acid Base
 disturbance are never enough to correct the change in
 pH , they only act to reduce the severity.

 Mixed Acid  Base Disorder – Presence of more than
 one acid base disorder simultaneously .
Normogram (for simple acid-base ds)
Characteristics of Primary ACID BASE
                   Disorders
PRIMARY       PRIMARY RESPONSES COMPENSATORY
DISORDER      H+ ion pH    Primary RESPONSES
              Conc.        Defect
Metabolic                               PCO2
Acidosis         H+   pH   HCO3        Alveolar
                                    Hyperventilation
Metabolic                               PCO2
Alkalosis        H+   pH   HCO3       Alveolar
                                    Hypoventilation
Respiratory
Acidosis         H+   pH    PCO2        HCO3

Respiratory
Alkalosis        H+   pH    PCO2        HCO3
Compensation
Metabolic Disorders – Compensation in these disorders leads to
a change in PCO2



 METABOLIC
  ACIDOSIS • For every 1mmol/l in HCO3 the
             PCO2 falls by 1.25 mm Hg



 METABOLIC
 ALKALOSIS        • For every 1mol/l in HCO3 the
                    PCO2 by 0.75 mm Hg
In Respiratory Disorders
    PCO2              Kidney              HCO3 Reabsorption
  Compensation begins to appear in 6 – 12 hrs and is fully
  developed only after a few days.

1.ACUTE
Before the onset of compensation
Resp. acidosis – 1mmHg in PCO2 HCO3 by 0.1meq/l
Resp. alkalosis – 1mmHg in PCO2 HCO3 by 0.2 meq/l

2.CHRONIC (>24 hrs)
After compensation is fully developed
Resp. acidosis – 1mmHg in PCO2          HCO3 by 0.4meq/l
Resp. alkalosis – 1mmHg in PCO2          HCO3 by 0.4meq/l
Body’s physiologic response to Primary disorder
in order to bring pH towards NORMAL limit


Full compensation
Partial compensation
No compensation…. (uncompensated)

BUT never overshoots,
If a overshoot pH is there,
Take it granted it is a MIXED disorder
STEP WISE APPROACH
          to
   Interpretation Of
     ABG reports
Normal Values
ANALYTE        Normal Value   Units

   pH           7.35 - 7.45

  PCO2           35 – 45      mm Hg

  PO2            80 – 100     mm Hg`

 [HCO3]          22 – 26      meq/L

  SaO2            95-100        %

Anion Gap         10 + 2      meq/L

 ∆HCO3           +2 to -2     meq/L
STEP 0   • Is this ABG Authentic?

STEP 1 • ACIDEMIA or ALKALEMIA?

         • RESPIRATORY or METABOLIC?
STEP 2

STEP 3 • Is COMPENSATION adequate?

STEP 4 • If METABOLIC – ANION GAP?

       • If High gap Metabolic Acidosis–
STEP 5   GAP GAP?
Step 0 – Authentic or Not?
      Verify that the ABG values are internally
    accurate.
    ◦ The accuracy of the values can be established
      by confirming that they satisfy a simplified form
      of the Henderson-Hasselbalch equation:
        [H+](nmol/L) = 24 × pCO2(mm Hg) ∕ [HCO3-]
                           (mEq/L)

    ◦ [H+] = 10 exp(-pH). Within the pH range of 7.26
      to 7.45 [H+] in nmol/L = 80 − the decimal of the
                              pH
           (e.g., for pH = 7.25, [H+] = 80 − 25 = 55
Example: using Kassirer-
    Bleich equation
     [H+](nmol/L) = 24 × pCO2(mm Hg) ∕ [HCO3-]
                      (mEq/L)
 ABG: pH = 7.25, pCO2 = 30, HCO3- = 22

                                    30
                    55 = 24 x
                                    22
                      55 ≠ 32
                 LAB ERROR
STEP 1          ACIDEMIA OR ALKALEMIA?



   Look at pH
             <7.35 - acidemia
             >7.45 – alkalemia

     RULE – An acid base abnormality is present even if
      either the pH or PCO2 are Normal.
STEP 2    RESPIRATORY or METABOLIC?

IS PRIMARY DISTURBANCE RESPIRATORY OR
METABOLIC?


pH   HCO3 or        pH HCO3              METABOLIC

pH   PCO2 or pH          PCO2           RESPIRATORY

  RULE- If either the pH or PCO2 is Normal, there is a
  mixed metabolic and respiratory acid base disorder.
Step 3 Is Compensation
Adequate?
Disorder    Prediction of Compensation            pH     HCO3–   PaCO2




Metabolic   PaCO2 will 1.25 mmHg per mmol/L in
acidosis    [HCO3-]                               Low    Low     Low




Metabolic    PaCO2 will 0.75 mmHg per mmol/L in   High   High    High
alkalosis   [HCO3-]
Disorder      Prediction of Compensation            pH     HCO3–   PaCO2

Respiratory                                         High   Low     Low
alkalosis
Acute         [HCO3-] will 0.2 mmol/L per mmHg in
              PaCO2


 Chronic      [HCO3-] will 0.4 mmol/L per mmHg in
              PaCO2


Respiratory                                         Low    High    High
acidosis
Acute         [HCO3-] will 0.1 mmol/L per mmHg in
              PaCO2


 Chronic      [HCO3-] will 0.4 mmol/L per mmHg in
              PaCO2
STEP 0   • Is this ABG Authentic?

STEP 1   • ACIDEMIA or ALKALEMIA?
         • RESPIRATORY or METABOLIC?
STEP 2

STEP 3 • If Respiratory – ACUTE or CHRONIC?
STEP 4 • Is COMPENSATION adequate?

STEP 4   • If METABOLIC – ANION GAP?

         • If High gap Metabolic Acidosis–
STEP 6     GAP GAP?
Electrochemical Balance in Blood


100%      UC
                   UA
 90%
 80%               HCO3
         Na                 Sulfate
 70%                        Phosphate
 60%                        Mg- OA
                    Cl
 50%                        K - Proteins
 40%                        Ca-HCO3
 30%                        Na- Cl
 20%
 10%
  0%
       CATIONS   ANIONS
Anion Gap
AG based on principle of electroneutrality:

 Total Serum Cations = Total Serum Anions
 M cations + U cations = M anions + U anions
 Na + (K + Ca + Mg)       = HCO3 + Cl + (PO4 + SO4
                                     + Protein + Organic Acids)
 Na + UC                 = HCO3 + Cl + UA
 But in Blood there is a relative abundance of Anions, hence
             Anions       > Cations
 Na – (HCO3 + Cl)        = UA – UC
 Na – (HCO3 + Cl)        = Anion Gap
METABOLIC ACIDOSIS-
  STEP 4
                          ANION GAP?

IN METABOLIC ACIDOSIS WHAT IS THE ANION GAP?
       ANION        GAP(AG) = Na – (HCO3 + Cl)
                     Normal Value = 10 + 2

Adjusted Anion Gap = Observed AG +2.5(4.5- S.Albumin)
50%    in S. Albumin     75% in Anion Gap !!!

                          High Anion Gap Metabolic Acidosis
Metabolic Acidosis
                          Normal Anion Gap Acidosis
STEP 5            CO EXISTANT METABOLIC
                     DISORDER – “Gap Gap‖?

C/O HGAG METABOLIC ACIDOSIS,ANOTHER DISORDER?
 ∆ Anion Gap = Measured AG – Normal AG
                   Measured AG – 12

∆ HCO3 = Normal HCO3 – Measured HCO3
                24 – Measured HCO3
Ideally,      ∆Anion Gap = ∆HCO3
For each 1 meq/L increase in AG, HCO3 will fall by 1 meq/L

∆AG/ HCO3- = 1  Pure High AG Met Acidosis
 AG/ HCO3- > 1  Assoc Metabolic Alkalosis
 AG/ HCO3- < 1  Assoc N AG Met Acidosis
CLINICAL CASE
  SCENARIO
Case Scenario
A patient with a severe postoperative
ileus requires the insertion of a
nasogastric (NG) tube for
decompression. After several days on
the floor, he develops a line infection and
is moved to the ICU once he becomes
pressor dependent. The patient's ABG
reveals a pH = 7.44, pCO2 = 12, and
[HCO3-] = 8. The [Na+] = 145 with [Cl-] =
102.
ABG: pH = 7.44, pCO2 = 12, [HCO3-] = 8,
 [Na+] = 145 with [Cl-] = 102

◦ With knowledge of the common clinical
  scenarios leading to acid-base
  disturbances, this patient is at risk for
  developing a metabolic alkalosis from NG
  suction and a metabolic acidosis and
  respiratory alkalosis from sepsis.
◦ Step 1. [H+] = 80 − 44 = 36.
   Does 36 = 24 × 12 / 8? It does.
◦ Step 2. The patient is mildly alkalemic, which
  can be explained by the low pCO2 but not by
  the low [HCO3-], suggesting that a respiratory
  alkalosis may be the primary derangement.
ABG: pH = 7.44, pCO2 = 12, [HCO3-] = 8,
 [Na+] = 145 with [Cl-] = 102
◦ Step 3
   a. The drop in [HCO3-] might be an appropriate
    compensation for a chronic respiratory alkalosis
    ([HCO3-] is 0.4 mmol/L per mmHg in PaCO2)
    However, 24 − [(40 − 12) × 0.4] = 12.8, which is not
    close to the observed [HCO3-] of 8. A mixed disorder is
    implied.
   b. AG = 145 − 102 − 8 = 35. There is an elevated
    AG, suggests a concomitant metabolic acidosis.
   c. Delta anion gap=35 − 10 = 25.
   d. Delta bicarbonate= 24 − 8 =16.
   Ratio of above two is more than 1 suggestive of
    associated metabolic alkalosis. This has proven to be
    a triple acid-base disorder with an elevated anion
    gap acidosis, a metabolic alkalosis, and a
    respiratory alkalosis, as was alluded to in Step 1
Metabolic Acidosis
   Metabolic acidosis can occur because of
    ◦ increase in endogenous acid production (lactate and ketoacids)
    ◦ loss of bicarbonate (as in diarrhea)
    ◦ accumulation of endogenous acids (as in renal failure).
   Effects on the respiratory, cardiac, and nervous
    systems.
    ◦ Kussmaul respiration
    ◦ Intrinsic cardiac contractility may be depressed, but inotropic
      function can be normal because of catecholamine release.
    ◦ Both peripheral arterial vasodilation and central
      venoconstriction can be present
    ◦ The decrease in central and pulmonary vascular compliance
      predisposes to pulmonary edema with even minimal volume
      overload.
    ◦ Central nervous system function is depressed, with
      headache, lethargy, stupor, and, in some cases, even coma.
Metabolic Acidosis: Essentials of
Diagnosis
 Decreased HCO3– with acidemia.
 Classified into high anion gap acidosis
  and normal anion gap (hyperchloremic)
  acidosis.
 The high anion gap acidoses are seen in
  lactic acidosis, ketoacidosis, renal failure
  or toxins.
 Normal anion gap acidosis is mainly
  caused by gastrointestinal HCO3– loss or
  RTA.
    Urinary anion gap may help distinguish
     between these causes.
Causes of High-Anion-Gap
     Metabolic Acidosis
1.       Lactic acidosis
            Poor tissue perfusion (type A)
            Aerobic disorders (type B)
2.       Ketoacidosis
          Diabetic
          Alcoholic
          Starvation
3.       Toxins
            Ethylene glycol
            Methanol
            Salicylates
            Propylene glycol: as vehicle of medications
            Pyroglutamic acid: acetaminophen toxicity
4.       Renal failure (acute and chronic)
Ketacidosis –
    Diabetic Ketoacidosis (DKA):
 DKA is caused by increased fatty acid
  metabolism and the accumulation of ketoacids
  (acetoacetate and -hydroxybutyrate).
 DKA usually occurs in insulin-dependent
  diabetes mellitus in association with
    cessation of insulin or
    an intercurrent illness, such as an
     infection, gastroenteritis, pancreatitis, or
     myocardial infarction, which increases insulin
     requirements temporarily and acutely.
 The accumulation of ketoacids accounts for the
  increment in the Anion Gap and is
  accompanied most often by hyperglycemia
  [glucose > 17 mmol/L (300 mg/dL)].
Glucose,a mmol/L (mg/dL)    13.9–33.3 (250–600)

Sodium, meq/L               125–135
Potassiuma                  Normal to ↑

Magnesiuma                  Normal ( plasma levels may be normal or high at
                            presentation, total-body stores are usually
                            depleted)
Chloridea                   Normal

Phosphatea                  ↓

Creatinine                  Slightly ↑
Osmolality (mOsm/mL)        300–320
Plasma ketonesa             ++++

Serum bicarbonate,a meq/L <15 meq/L

Arterial pH                 6.8–7.3
Arterial PCO2,a mmHg        20–30

Anion gapa[Na - (Cl + HCO3)] ↑
Management
   Confirm diagnosis (plasma glucose, positive serum
    ketones, metabolic acidosis).
   Admit to hospital; intensive-care setting may be necessary for
    frequent monitoring or if pH < 7.00 or unconscious.
   Assess:
     Serum electrolytes (K+, Na+, Mg2+, Cl-, bicarbonate, phosphate)
     Acid-base status—pH, HCO3-, PCO2, b-hydroxybutyrate
     Renal function (creatinine, urine output)
   Replace fluids:
     2–3 L of 0.9% saline over first 1–3 h (10–15 mL/kg per hour);
     subsequently, 0.45% saline at 150–300 mL/h;
     change to 5% glucose and 0.45% saline at 100–200 mL/h when
      plasma glucose reaches 250 mg/dL (14 mmol/L).
   Administer short-acting insulin:
     IV (0.1 units/kg) or IM (0.3 units/kg), then 0.1 units/kg per hour by
      continuous IV infusion;
     increase 2- to 3-fold if no response by 2–4 h.
     If initial serum potassium is < 3.3 mmol/L (3.3 meq/L), do not
      administer insulin until the potassium is corrected to > 3.3 mmol/L
Management
 Assess patient: What precipitated the episode
  (noncompliance, infection, trauma, infarction, cocaine)?
  Initiate appropriate workup for precipitating event
  (cultures, CXR, ECG).
 Measure
     capillary glucose every 1–2 h;
     electrolytes (especially K+, bicarbonate, phosphate) and anion
      gap every 4 h for first 24 h.
 Monitor blood pressure, pulse, respirations, mental
  status, fluid intake and output every 1–4 h.
 Replace K+:
     10 meq/h when plasma K+ < 5.5 meq/L, ECG normal, urine flow
      and normal creatinine documented;
     administer 40–80 meq/h when plasma K+ < 3.5 meq/L or if
      bicarbonate is given.
 Continue above until patient is stable, glucose goal is 150–
  250 mg/dL, and acidosis is resolved. Insulin infusion may be
  decreased to 0.05–0.1 units/kg per hour.
 Administer intermediate or long-acting insulin as soon as
  patient is eating. Allow for overlap in insulin infusion and
 since insulin prevents production of
  ketones, bicarbonate therapy is rarely
  needed except with extreme acidemia (pH <
  7.1), and then in only limited amounts.
 Patients with DKA are typically volume
  depleted and require fluid resuscitation with
  isotonic saline.
 Volume overexpansion is not uncommon
  after IV fluid administration, and contributes
  to the development of a hyperchloremic
  acidosis during treatment of DKA because
  volume expansion increases urinary
  ketoacid anion excretion (loss of potential
  bicarbonate).
 The mainstay for treatment of this condition
  is IV regular insulin
Drug, Toxin-Induced Acidosis
 Plasma osmolality is calculated according to the following
  expression:
           Posm = 2Na+ + Glu + BUN (all in mmol/L)
 Posm = 2Na+ + Glu/18 + BUN/2.8 (milligrams per deciliter)
 The calculated and determined osmolality should be within
  10–15 mmol/kg H2O
 When the measured osmolality exceeds the calculated
  osmolality by >15–20 mmol/kg H2O, either prevails.
    Either the serum sodium is spuriously low, as with
     hyperlipidemia or hyperproteinemia
     (pseudohyponatremia)
    Osmolytes other than sodium salts, glucose, or urea
     have accumulated in plasma. Examples include
     mannitol, radiocontrast media, isopropyl
     alcohol, ethylene glycol, propylene
   In this situation, the difference between the
    calculated osmolality and the measured osmolality
    (osmolar gap) is proportional to the concentration of
    the unmeasured solute.
   Alcohols: With an appropriate clinical history and index
    of suspicion, identification of an osmolar gap is helpful
    in identifying the presence of poison-associated AG
    acidosis. Three alcohols may cause fatal intoxications:
    ethylene glycol, methanol, and isopropyl alcohol

   Salicylates: Salicylate intoxication in adults usually
    causes respiratory alkalosis or a mixture of high-AG
    metabolic acidosis and respiratory alkalosis. Only a
    portion of the AG is due to salicylates. Lactic acid
    production is also often increased
Renal failure Acidosis
   The hyperchloremic acidosis of moderate renal
    insufficiency is eventually converted to the
    high-AG acidosis of advanced renal failure.

   At GFRs below 20 mL/min, the inability to
    excrete H+ with retention of acid anions such
    as PO43– and SO42– results in an increased
    anion gap acidosis

 [HCO3–] rarely falls to <15 mmol/L, and the AG
  is rarely >20 mmol/L, indicating that the acid
  retained in chronic renal disease is buffered by
  alkaline salts from bone.
 Results in significant loss of bone mass due to
  reduction in bone calcium carbonate and
  increases urinary calcium excretion.
Treatment of high anion gap
acidosis
 Treatment is aimed at the underlying disorder, such as
  insulin and fluid therapy for diabetes and appropriate
  volume resuscitation to restore tissue perfusion. The
  metabolism of lactate will produce HCO3– and increase
  pH.
 Controversy     regarding administration of large
  amounts of HCO3–
     may have deleterious effects, including hypernatremia and
      hyperosmolality.
     Intracellular pH may decrease because administered HCO3–
      is converted to CO2, which easily diffuses into cells, combines
      with water to create additional hydrogen ions and worsening
      of intracellular acidosis and this could impair cellular function
     Alkali      administration     is     known      to      stimulate
      phosphofructokinase activity, thus exacerbating lactic acidosis
      via enhanced lactate production. Ketogenesis is also
      augmented by alkali therapy.
Treatment of high anion gap
       acidosis…..
     ◦ In salicylate intoxication, alkali therapy must be started unless blood
       pH is already alkalinized by respiratory alkalosis, since the
       increment in pH converts salicylate to more impermeable salicylic
       acid and thus prevents central nervous system damage.
     ◦ In DKA, alkali must be administered in extreme alkalemia (pH<7.1)
     ◦ In alcoholic ketoacidosis, thiamine should be given together with
       glucose to avoid the development of Wernicke's encephalopathy.
    The amount of HCO3– deficit can be calculated as follows:
    Amount of HCO3– deficit = 0.5 X body weight X (24 - HCO3–)
     ◦ Half of the calculated deficit should be administered within the first
       3–4 hours to avoid overcorrection and volume overload.
    In methanol intoxication
     ◦ ethanol has been used as a competitive substrate for alcohol
       dehydrogenase, which metabolizes methanol to formaldehyde
     ◦ or through direct inhibition of alcohol dehydrogenase by fomepizole
Hyperchloremic (Nongap)
Metabolic Acidoses: causes
 I. Gastrointestinal bicarbonate loss
  Diarrhea
  External pancreatic or small-bowel drainage
  Ureterosigmoidostomy, jejunal loop, ileal loop
  Drugs - Calcium chloride , Magnesium sulfate
   (diarrhea), Cholestyramine (bile acid diarrhea)
  II.   Renal acidosis
  Hypokalemia
     Proximal RTA (type 2)
     Distal (classic) RTA (type 1)
  Hyperkalemia
     Generalized distal nephron dysfunction (type 4 RTA)
         a. Mineralocorticoid deficiency
         b. Mineralocorticoid resistance (autosomal dominant PHA I)
         c. Voltage defect (autosomal dominant PHA I and PHA II)
         d. Tubulointerstitial disease
III. Drug-induced hyperkalemia (with renal insufficiency)
  ◦ Potassium-sparing diuretics
    (amiloride, triamterene, spironolactone)
  ◦ Trimethoprim
  ◦ Pentamidine
  ◦ ACE-Is and ARBs
  ◦ Nonsteroidal anti-inflammatory drugs
  ◦ Cyclosporine and tacrolimus

IV. Other
   ◦ Acid loads (ammonium chloride, hyperalimentation)
   ◦ Loss of potential bicarbonate: ketosis with ketone excretion
   ◦ Expansion /dilutional acidosis (rapid saline administration)
   ◦ Hippurate
   ◦ Cation exchange resins
Approach: Urinary Anion Gap
   Increased renal NH4+Cl– excretion to enhance H+ removal is a normal
    physiologic response to metabolic acidosis. NH3 reacts with H+ to
    form NH4+, which is accompanied by the anion Cl– for excretion.
   Urinary anion gap from a random urine sample ([Na++ K+]– Cl–)
    reflects the ability of the kidney to excrete NH4Cl as in the following
    equation:
              Na+ + K+ + NH4+ = Cl– + 80
       urinary anion gap is equal to (80 – NH4+)
   Gastrointestinal HCO3– loss (diarrhea), the renal acidification ability
    remains normal and NH4Cl excretion increases in response to the
    acidosis. urinary anion gap is negative (eg, –30 mEq/L).
   Distal RTA, the urinary anion gap is positive (eg, +25 mEq/L), since
    the basic lesion in the disorder is the inability of the kidney to excrete
    H+ and thus the inability to increase NH4Cl excretion.
   Proximal (type II) RTA, the kidney has defective HCO3–
    reabsorption, leading to increased HCO3– excretion rather than
    decreased NH4Cl excretion. Thus, the urinary anion gap is negative
   Urinary pH may not as readily differentiate
    between the two causes because volume
    depletion or potassium depletion, which can
    accompany diarrhea (and surreptitious laxative
    abuse) may impair renal acidification.
   Thus, when volume depletion is present, the
    urinary anion gap is a better measurement of
    ability to acidify the urine than urinary pH.
   When large amounts of other anions are present
    in the urine, the urinary anion gap may not be
    reliable. In such a situation, NH4+ excretion can
    be estimated using the urinary osmolar gap.
   NH4+ excretion (mmol/L) = 0.5 x Urinary osmolar
    gap = 0.5 [U osm – 2(U Na++U K+) + U urea + U
    glucose]
   where urinary (U) concentrations and osmolality
    are in millimoles per liter.
Renal        Serum   Urinary     Titratable Urinary   Treatment
                  Defect       [K+]    NH4+ Plus   Acid       Anion
                                       Minimal                Gap
                                       Urine pH
Gastrointestinal None          ↓       < 5.5       ↑↑        Negative Na+, K+, and
HCO3– loss                                                            HCO3– as
                                                                      required
Renal tubular
acidosis
 I. Classic distal Distal H+   ↓       > 5.5       ↓         Positive   NaHCO3 (1–3
                   secretion                                            mEq/kg/d)
 II. Proximal     Proximal     ↓       < 5.5       Normal    Negative NaHCO3 or
secretion         HCO3–                                               KHCO3 (10–15
                  abspn                                               mEq/kg/d),
                                                                      thiazide
 IV.            Distal Na+   ↑         < 5.5       ↓         Positive   Fludrocortisone
Hyporeninemic reabsorption                                              (0.1–0.5 mg/d),
                 +                                                      dietary K+ rstrn,
hypoaldosteroni K secretion,
                and H+                                                  furosemide (40–
sm
                  secretion                                             160 mg/d),
                                                                        NaHCO3 (1–3
                                                                        mEq/kg/d)
Treatment of normal Anion gap
     Acidosis
   Gastrointestinal: Correction of the contracted ECFV with NaCl
  and repair of K+ deficits corrects the acid-base disorder, and
  chloride deficiency.
 RTA : administration of alkali (either as bicarbonate or citrate) to
  correct metabolic abnormalities and prevent nephrocalcinosis and
  renal failure.
  ◦ Proximal RTA : Large amounts of alkali (10–15 mEq/kg/d) may
    be required to treat proximal RTA because most of the alkali is
    excreted into the urine, which exacerbates hypokalemia. Thus, a
    mixture of sodium and potassium salts, such as K-Shohl
    solution, is preferred. The addition of thiazides may reduce the
    amount of alkali required, but hypokalemia may develop.
  ◦ Distal RTA : Correction of type 1 distal RTA requires a smaller
    amount of alkali (1–3 mEq/kg/d) and potassium supplementation
    as needed.
  ◦ Type IV RTA: dietary potassium restriction may be needed and
    potassium-retaining drugs should be withdrawn. Fludrocortisone
    may be effective in cases with hypoaldosteronism, but should be
    used with care, preferably in combination with loop diuretics.
    alkali supplementation (1–3 mEq/kg/d) may be required.
Metabolic Alkalosis

   Essentials of Diagnosis
    ◦ High HCO3– with alkalemia.
    ◦ Evaluate effective circulating volume by
      physical examination and check urinary
      chloride concentration. This will help
      differentiate saline-responsive metabolic
      alkalosis from saline-unresponsive
      alkalosis
Metabolic Alkalosis
 Occurs as a result of net gain of [HCO3–] or loss
  of nonvolatile acid (usually HCl by vomiting) from
  the extracellular fluid.
 The disorder involves
     a generative stage, in which the loss of acid usually
      causes alkalosis
     maintenance stage, in which the kidneys fail to
      compensate by excreting HCO3–.
   Classified based on "saline responsiveness" or
    urinary Cl–, which are markers for volume status
     Saline-responsive metabolic alkalosis is a sign
      of extracellular volume contraction
     Saline-unresponsive alkalosis implies a volume-
      expanded state
Symptoms
 With metabolic alkalosis, changes in central and
  peripheral nervous system function are similar to
  those of hypocalcemia : symptoms include
    Mental confusion
    Obtundation
    Predisposition to seizures
    Paresthesia
    Muscular cramping
    Tetany
    Aggravation of arrhythmias
 Hypoxemia in chronic obstructive pulmonary
  disease.
 Related electrolyte abnormalities include
     Hypokalemia- Weakness and hyporeflexia
     hypophosphatemia.
Classification and etiology
       I. Saline-Responsive (UCl < 10 mEq/d)
 Excessive body bicarbonate content
    Renal alkalosis
      Diuretic therapy
      Poorly reabsorbable anion therapy:
       carbenicillin, penicillin, sulfate, phosphate
      Posthypercapnia
    Gastrointestinal alkalosis
      Loss of HCl from vomiting or nasogastric suction
      Intestinal alkalosis: chloride diarrhea
    Exogenous alkali
      NaHCO3 (baking soda)
      Sodium citrate, lactate, gluconate, acetate
      Transfusions
      Antacids
 Normal body bicarbonate content
    "Contraction alkalosis"
II. Saline-Unresponsive (UCl > 10 mEq/d)
   Excessive body bicarbonate content
     Renal alkalosis
       Normotensive
         Bartter's syndrome (renal salt wasting and
          secondary hyperaldosteronism)
         Severe potassium depletion
         Refeeding alkalosis
         Hypercalcemia and hypoparathyroidism
       Hypertensive
         Endogenous mineralocorticoids
         Primary aldosteronism
         Hyperreninism
         Adrenal enzyme deficiency: 11- and 17-
          hydroxylase
         Liddle's syndrome
         Exogenous mineralocorticoids
         Licorice
Treatment
   Mild alkalosis is generally well tolerated. Severe or
    symptomatic alkalosis (pH > 7.60) requires urgent
    treatment.
   Saline-Responsive Metabolic Alkalosis
    ◦ Aimed at correction of extracellular volume
      deficit.
    ◦ Depending on the degree of
      hypovolemia, adequate amounts of 0.9%
      NaCl and KCl should be administered.
    ◦ Discontinuation of diuretics and
      administration of H2-blockers in patients
      whose alkalosis is due to nasogastric suction
      can be useful.
    ◦ If impaired pulmonary or cardiovascular
      status prohibits adequate volume
      repletion, acetazolamide, 250–500 mg
      intravenously every 4–6 hours, can be used.
Treatment…..
     One must be alert to the possible development of
      hypokalemia, since potassium depletion can be
      induced by forced kaliuresis via bicarbonaturia.
     Administration of acid can be used as emergency
      therapy. HCl, 0.1 mol/L, is infused via a central vein
      (the solution is sclerosing). Dosage is calculated to
      decrease the HCO3– level by 50% over 2–4
      hours, assuming an HCO3– volume of distribution (L)
      of 0.5% body weight (kg).
     Patients with marked renal failure may require dialysis.

   Saline-Unresponsive Metabolic Alkalosis
     surgical removal of a mineralocorticoid-producing
      tumor
     blockage of aldosterone effect with an ACE inhibitor or
      with spironolactone.
     primary aldosteronism can be treated only with
      potassium repletion.
Respiratory Acidosis
Results from decreased alveolar ventilation and hypercapnia
both due to pulmonary and non pulmonary disorders
 Central                                 Neuromuscular
  ◦ Drugs                                  ◦ Poliomyelitis
     (anesthetics, morphine, sedatives)    ◦ Kyphoscoliosis
  ◦ Stroke                                 ◦ Myasthenia
  ◦ Infection                              ◦ Muscular dystrophies
 Airway                                  Miscellaneous
  ◦ Obstruction                            ◦ Obesity
  ◦ Asthma                                 ◦ Hypoventilation
 Parenchyma                               ◦ Permissive hypercapnia
  ◦ Emphysema
  ◦ Pneumoconiosis
  ◦ Bronchitis
  ◦ Adult respiratory distress
     syndrome
  ◦ Barotrauma
Respiratory Acidosis
   Acute respiratory failure
      associated with severe acidosis and only a small increase
       in the plasma bicarbonate.
      After 6–12 hours, the primary increase in PCO2 evokes a
       renal compensatory response to generate more HCO3–
       , which tends to ameliorate the respiratory acidosis. This
       takes several days to complete.
   Chronic respiratory acidosis
      seen in patients with underlying lung disease, such as
       chronic obstructive pulmonary disease.
      Urinary excretion of acid in the form of NH4+ and Cl– ions
       results in the characteristic hypochloremia of chronic
       respiratory acidosis.
      When chronic respiratory acidosis is corrected
       suddenly, especially in patients who receive mechanical
       ventilation, there is a 2- to 3-day lag in renal bicarbonate
       excretion, resulting in posthypercapnic metabolic
       alkalosis.
   Symptoms and Signs
    ◦ With acute onset, there is somnolence and confusion, and
      myoclonus with asterixis
    ◦ Coma from CO2 narcosis may ensue
    ◦ Severe hypercapnia increases cerebral blood flow and
      cerebrospinal fluid pressure. Signs of increased intracranial
      pressure (papilledema, pseudotumor cerebri) may be seen.
   Laboratory Findings
    ◦ Arterial pH is low
    ◦ PCO2 is increased
    ◦ Serum HCO3– is elevated, but not enough to completely
      compensate for the hypercapnia.
    ◦ If the disorder is chronic, hypochloremia is seen.
   Treatment
    ◦ In all forms of respiratory acidosis, treatment is directed at the
      underlying disorder to improve ventilation.
    ◦ Because opioid drug overdose is an important reversible cause of
      acute respiratory acidosis, naloxone, 0.04–2 mg intravenously is
      administered to all such patients if no obvious cause for
      respiratory depression is present.
    ◦ Mechanical ventilation for oxygenation till it restores back to
      normal maybe needed
Respiratory Alkalosis(Hypocapnia)

   Respiratory alkalosis, or hypocapnia, occurs when hyperventilation
    reduces the PCO2, which increases the pH
   Symptoms and Signs
     In acute cases (hyperventilation), there is light-headedness,
      anxiety, paresthesias, numbness about the mouth, and a tingling
      sensation in the hands and feet. Tetany occurs in more severe
      alkalosis from a fall in ionized calcium.
     In chronic cases, findings are those of the responsible condition.
   Laboratory Findings
     Arterial blood pH is elevated, and PCO2 is low. Serum
      bicarbonate is decreased in chronic respiratory alkalosis.
     Determination of appropriate compensatory changes in the
      HCO3– is useful to sort out the presence of an associated
      metabolic disorder
       the changes in HCO3– values are greater if the respiratory
        alkalosis is chronic
       Although serum HCO3– is frequently below 15 mEq/L in
        metabolic acidosis, it is unusual to see such a low level in
        respiratory alkalosis, and its presence would imply a
        superimposed (noncompensatory) metabolic acidosis.
Causes of respiratory alkalosis.

   Hypoxia :
     Decreased inspired oxygen tension
     High altitude
     Ventilation/perfusion inequality
     Hypotension
     Severe anemia
   CNS-mediated disorders
     Voluntary hyperventilation
       Anxiety-hyperventilation syndrome
     Neurologic disease:
       Cerebrovascular accident
        (infarction, hemorrhage)
       Infection
       Trauma
       Tumor
Causes…
     Pharmacologic and hormonal stimulation :
      Salicylates, Nicotine, Xanthines ,Pregnancy
      (progesterone)
     Hepatic failure
     Gram-negative septicemia
     Recovery from metabolic acidosis
     Heat exposure
   Pulmonary disease
     Interstitial lung disease
     Pneumonia
     Pulmonary embolism
     Pulmonary edema
   Mechanical overventilation
Treatment
   Treatment is directed toward the underlying cause.
   In acute hyperventilation syndrome from
    anxiety, reassurance , attention to underlying psychological
    stress and rebreathing into a paper bag will increase the
    PCO2.
      The processes are usually self-limited since muscle
        weakness caused by hyperventilation-induced alkalemia will
        suppress ventilation.
   Sedation may be necessary if the process persists however
    Antidepressants and sedatives should be avoided
   Adrenergic blockers may ameliorate peripheral
    manifestations of the hyperadrenergic state.
   Rapid correction of chronic respiratory alkalosis may result
    in metabolic acidosis as PCO2 is increased in the setting of
    previous compensatory decrease in HCO3–.
   If respiratory alkalosis complicates ventilator
    management, changes in dead space, tidal volume, and
    frequency can minimize the hypocapnia
Conclusion
   Clinical suspicion according to scenario
   Primary disorder from pH and bicarbonate or CO2
    values
   Degree of compensation…? Inappropriate? mixed
    disorder
   Anion gap (corrected for serum albumin
    change)…? Primary metabolic acidosis
   Corrected bicarbonate levels…?
    More than normal- associated metabolic alkalosis
    Less - associated metabolic non-anion gap acidosis
   Review compensation for metabolic ds…confirm if
    initially suspected respiratory disorder exists
   Metabolic acidosis: ..see AG
      High AG: …plasma osmolal gap
        more.. Toxins
        Less.. KA, RF, LA,
      Normal AG acidosis:
        urine AG….negative..GI
        urinary pH ….high..RTA 1
        serum potassium….high..RTA4

   Metabolic alkalosis:
    ◦ Urinary Chloride…
      Less..saline responsive…ECF contraction
      more ..saline non responsive
        Plasma Potassium…low..K depletion
        High…Blood pressure
        Plasma renin
1.PaCO2 equation:

              VCO2 x 0.863         VCO2 = CO2 production
     PaCO2   = -----------------           VA = VE – VD
                      VA                    VE = minute (total) ventilation
                                            VD = dead space ventilation
                                            0.863 converts units to mm Hg

                          Condition           State of
PaCO2                     in blood            alveolar ventilation
>45 mm Hg                 Hypercapnia        Hypoventilation
35 - 45 mm Hg             Eucapnia           Normal ventilation
<35 mm Hg                 Hypocapnia         Hyperventilation

PaCO2 reflects ratio of metabolic CO2 production to alveolar
 ventilation
Hypercapnia

                    VCO2 x 0.863
     PaCO2 =       ------------------
                             VA

 The only physiologic reason for elevated PaCO2 is
inadequate alveolar ventilation (VA) for the amount of the
body‘s CO2 production (VCO2). Since alveolar ventilation
(VA) equals minute ventilation (VE) minus dead space
ventilation (VD), hypercapnia can arise from insufficient
VE, increased VD, or a combination.
Hypercapnia
              VCO2 x 0.863
   PaCO2 =      ------------------
                  VA                 VA = VE – VD


 Examples of inadequate VE leading to decreased VA and
  increased PaCO2: sedative drug overdose; respiratory muscle
  paralysis; central hypoventilation

 Examples of increased VD leading to decreased VA and
  increased PaCO2: chronic obstructive pulmonary disease;
  severe pulmonary embolism, pulmonary edema.
Physiologic effects of
        hypercapnia
◦ 1) An elevated PaCO2 will lower the PAO2 (see Alveolar gas
  equation), and as a result lower the PaO2.

◦ 2) An elevated PaCO2 will lower the pH (see Henderson-
  Hasselbalch equation).

◦ 3) The higher the baseline PaCO2, the greater it will rise for a
  given fall in alveolar ventilation, e.g., a 1 L/min decrease in
  VA will raise PaCO2 a greater amount when baseline PaCO2
  is 50 mm Hg than when it is 40 mm Hg.
Assessment of Oxygenation

Status
2.Alveolar gas equation     Oxygenation(alv)


3.Oxygen content equation    Oxygenation(tissue)
Oxygenation
    -----XXXX Diagnostics-----            Parameters: /limitations
Blood        Gas       Report
328
Pt ID
               03:44
               3245 / 00
                             Feb 5 2006   O2 Content of blood:
                                          (Hb x1.34x O2 Sat + 0.003x Dissolved O2 )
Measured                  37.0 0C
pH             7.452                      Remember Hemoglobin
pCO2           45.1             mm Hg
pO2            112.3            mm Hg

Corrected                 38.6 0C         Oxygen Saturation:
pH             7.436
pCO2           47.6           mm Hg       ( remember this is calculated …error prone)
pO2            122.4          mm Hg

Calculated Data                           Alveolar / arterial gradient:
HCO3 act        31.2         mmol / L     ( classify respiratory failure)
HCO3 std        30.5          mmol / L
BE              6.6          mmol / L
O2 ct           15.8         mL / dl
O2 Sat         98.4           %           Arterial / alveolar ratio:
ct CO2         32.5          mmol / L
pO2 (A -a)     30.2          mm Hg       Proposed to be less variable
pO2 (a/A)      0.78
                                          Same limitations as A-a gradient
Entered Data
Temp               38.6     0C
FiO2               30.0    %
ct Hb              10.5    gm/dl
Alveolar Gas Equation

             PAO2 = PIO2 - 1.2 (PaCO2)
where PAO2 is the average alveolar PO2, and PIO2 is the partial pressure
of inspired oxygen in the trachea


           PIO2 = FIO2 (PB – 47 mm Hg)
FIO2 is fraction of inspired oxygen and PB is the barometric pressure. 47
mm Hg is the water vapor pressure at normal body temperature.
Alveolar Gas Equation
   If FIO2 and PB are constant, then as PaCO2 increases
    both PAO2 and PaO2 will decrease (hypercapnia causes
    hypoxemia).

   If FIO2 decreases and PB and PaCO2 are constant, both
    PAO2 and PaO2 will decrease.

   If PB decreases (e.g., with altitude), and PaCO2 and FIO2
    are constant, both PAO2 and PaO2 will decrease
    (mountain climbing causes hypoxemia).
P(A-a)O2
   P(A-a)O2 is the alveolar-arterial difference in partial pressure of
    oxygen. It is commonly called the ―A-a gradient‖. It results from
    gravity-related blood flow changes within the lungs (normal
    ventilation-perfusion imbalance).


   Normal P(A-a)O2 ranges from 5 to 25 mm Hg breathing room air
    (it increases with age). A higher than normal P(A-a)O2 means the
    lungs are not transferring oxygen properly from alveoli into the
    pulmonary capillaries. Except for right to left cardiac shunts, an
    elevated P(A-a)O2 signifies some sort of problem within the lungs.
Physiologic causes of low PaO2
NON-RESPIRATORY                      P(A-a)O2
 Cardiac right to left shunt         Increased
 Decreased PIO2                              Normal

RESPIRATORY
 Pulmonary right to left shunt              Increased
 Ventilation-perfusion imbalance            Increased
 Diffusion barrier                          Increased
 Hypoventilation (increased PaCO2)          Normal
Ventilation-Perfusion imbalance
   A normal amount of ventilation-perfusion (V-Q)
    imbalance accounts for the normal P(A-a)O2.
   By far the most common cause of low PaO2 is an
    abnormal degree of ventilation-perfusion imbalance
    within the hundreds of millions of alveolar-capillary
    units. Virtually all lung disease lowers PaO2 via V-Q
    imbalance, e.g., asthma, pneumonia, atelectasis,
    pulmonary edema, COPD.
   Diffusion barrier is seldom a major cause of low PaO2
    (it can lead to a low PaO2 during exercise).
SaO2 and oxygen content

How much oxygen is in the blood? Oxygen content = CaO2 (mlO2/dl).

    CaO2 = quantity O2 bound          +    quantity O2 dissolved
            to hemoglobin                            in plasma

    CaO2 = (Hb x 1.34 x SaO2) +           (.003 x PaO2)


   Hb = hemoglobin in gm%; 1.34 = ml O2 that can be bound to each gm of
    Hb; SaO2 is percent saturation of hemoglobin with oxygen; .003 is
    solubility coefficient of oxygen in plasma: .003 ml dissolved O2/mm Hg
    PO2.
   Given arterial oxygen saturation (SpO2) =
    100%, Hb = 15 g/100 ml and arterial partial
    pressure of oxygen (PaO2) = 13.3 kPa, then
    the oxygen content of arterial blood (CaO2) is:
  CaO2 = 20.1 +0.3 = 20.4 ml/100 ml
 Similarly the oxygen content of mixed venous
  blood can be calculated. Given normal
  values of mixed venous oxygen saturation
  (SvO2) = 75% and venous partial pressure of
  oxygen (PvO2) = 6 kPa, so:
    CvO2 = 15.2 + 0.1 = 15.2 ml/100 ml
Oxygen dissociation curve: SaO2 vs. PaO2
Also shown are CaO2 vs. PaO2 for two different hemoglobin contents: 15 gm%
and 10 gm%. CaO2 units are ml O2/dl. P50 is the PaO2 at which SaO2 is 50%.
SaO2 – is it calculated or measured?

  SaO2 is measured in a ‗co-oximeter‘. The traditional ‗blood gas
  machine‘ measures only pH, PaCO2 and PaO2,, whereas the co-
  oximeter measures SaO2, carboxyhemoglobin, methemoglobin and
  hemoglobin content. Newer ‗blood gas‘ consoles incorporate a co-
  oximeter, and so offer the latter group of measurements as well as
  pH, PaCO2 and PaO2.

   Always make sure the SaO2 is measured, not calculated. If it is
  calculated from the PaO2 and the O2-dissociation curve, it
  provides no new information, and could be inaccurate --
  especially in states of CO intoxication or excess
  methemoglobin. CO and metHb do not affect PaO2, but do lower
  the SaO2.
Carbon monoxide – an important cause
    of hypoxemia
   Normal %COHb in the blood is 1-2%, from metabolism and small
    amount of ambient CO (higher in traffic-congested areas)
    All smokers have excess CO in their blood.
   CO binds @ 200x more avidly to hemoglobin than O2, displacing O2
    from the heme binding sites.
   CO : 1) decreases SaO2 by the amount of %COHb present, and 2)
    shifts the O2-dissociation curve to the left, retarding unloading of
    oxygen to the tissues.
   CO does not affect PaO2, only SaO2. To detect CO
    poisoning, SaO2 and/or COHb must be measured (requires co-
    oximeter). In the presence of excess CO, SaO2 (when measured)
    will be lower than expected from the PaO2.
CO does not affect PaO2!

A patient presented to the ER with headache and dyspnea.

   His first blood gases showed PaO2 80 mm Hg, PaCO2 38 mm Hg, pH
    7.43. SaO2 on this first set was calculated from the O2-dissociation
    curve at 97%, and oxygenation was judged normal.
    He was sent out from the ER and returned a few hours later with mental
    confusion; this time both SaO2 and COHb were measured (SaO2 shown
    by ‗X‘): PaO2 79 mm Hg, PaCO2 31 mm Hg, pH 7.36, SaO2
    53%, carboxyhemoglobin 46%.


     CO poisoning was missed on the first set of blood gases
                because SaO2 was not measured!
Causes of Hypoxia
1. Hypoxemia (=low PaO2 and/or low CaO2)
   ◦ a. reduced PaO2 – usually from lung disease (most common
     physiologic mechanism: V-Q imbalance)
   ◦ b. reduced SaO2 -- most commonly from reduced PaO2; other
     causes include carbon monoxide
     poisoning, methemoglobinemia, or rightward shift of the O2-
     dissociation curve
   ◦ c. reduced hemoglobin content -- anemia
2. Reduced oxygen delivery to the tissues
   ◦ a. reduced cardiac output -- shock, congestive heart failure
   ◦ b. left to right systemic shunt (as may be seen in septic shock)
3. Decreased tissue oxygen uptake
   ◦ a. mitochondrial poisoning (e.g., cyanide poisoning)
   ◦ b. left-shifted hemoglobin dissociation curve (e.g., from acute
     alkalosis, excess CO, or abnormal hemoglobin structure)
Arterial Oxygen Tension (PaO2)
   Normal value in healthy adult breathing
    room air at sea level 97 mm Hg.
     progressively with age
   Dependant upon
        1. FiO2
        2. Patm
   Hypoxemia is PaO2 < 80 mm Hg at RA
   Most pts who need ABG usually req O2
    therapy
   O2 therapy should not be
    withheld/interrupted ‗to determine PaO2
    on RA‘
Acceptable PaO2 Values on Room
Air
   Age Group            Accepable PaO2
                        (mm Hg)
   Adults upto 60 yrs   > 80
   & Children
   Newborn              40-70
   70 yrs               > 70
   80 yrs               > 60
   90 yrs               > 50

    60 yrs   80 mm Hg          1mm Hg/yr
Inspired O2 – PaO2 Relationship
   FIO2 (%)          Predicted Min
                     PaO2 (mm Hg)
   30                150
   40                200
   50                250
   80                400
   100               500

 If PaO2 < FIO2 x 5, pt probably hypoxemic at RA
Hypoxemia on O2 therapy
 Uncorrected: PaO2 < 80 mm Hg
     (< expected on RA & FIO2)
 Corrected: PaO2 = 80-100 mm Hg
     (= expected on RA but < expected for FIO2)
 Excessively Corrected: PaO2 > 100 mm Hg
     (> expected on RA but < expected for FIO2)
 PaO2 > expected for FIO2:
     1. Error in sample/analyzer
     2. Pt‘s O2 consumption reduced
     3. Pt does not req O2 therapy (if 1 & 2 NA)
Thanks

More Related Content

What's hot

Abg by dr girish
Abg by dr girishAbg by dr girish
Abg by dr girishGirish jain
 
ABG (Emergency Medicine)
ABG (Emergency Medicine)ABG (Emergency Medicine)
ABG (Emergency Medicine)kalyan ram
 
Abg Interpretation
Abg InterpretationAbg Interpretation
Abg Interpretationlaciecrone
 
SIMPLE AND SYSTEMATIC APPROACH TO Acid base disorders
SIMPLE AND SYSTEMATIC APPROACH TO Acid base disorders  SIMPLE AND SYSTEMATIC APPROACH TO Acid base disorders
SIMPLE AND SYSTEMATIC APPROACH TO Acid base disorders aishwaryajoshi18
 
ABG INTERPRETATION.pptx
ABG INTERPRETATION.pptxABG INTERPRETATION.pptx
ABG INTERPRETATION.pptxakash chauhan
 
Basics In Arterial Blood Gas Interpretation
Basics In Arterial Blood Gas InterpretationBasics In Arterial Blood Gas Interpretation
Basics In Arterial Blood Gas Interpretationgueste36950a
 
Abg.2 Arterial blood gas analysis and example interpretation
Abg.2 Arterial blood gas analysis and example interpretationAbg.2 Arterial blood gas analysis and example interpretation
Abg.2 Arterial blood gas analysis and example interpretationsamirelansary
 
Arterial Blood Gases Talk
Arterial Blood Gases TalkArterial Blood Gases Talk
Arterial Blood Gases TalkDang Thanh Tuan
 
ABG , ARTERIAL BLOOD GAS
ABG , ARTERIAL BLOOD GAS ABG , ARTERIAL BLOOD GAS
ABG , ARTERIAL BLOOD GAS raadqu12345678
 
Interpreting Blood Gases, Practical and easy approach
Interpreting Blood Gases, Practical and easy approachInterpreting Blood Gases, Practical and easy approach
Interpreting Blood Gases, Practical and easy approachMuhammad Asim Rana
 
Arterial Blood Gases Analysis
Arterial Blood Gases AnalysisArterial Blood Gases Analysis
Arterial Blood Gases AnalysisGamal Agmy
 
Community acquired pneumonia
Community acquired pneumoniaCommunity acquired pneumonia
Community acquired pneumoniaAbhay Mange
 
Acid base disorders - acidosis alkalosis metabolic respiratory
Acid base disorders -  acidosis alkalosis metabolic respiratoryAcid base disorders -  acidosis alkalosis metabolic respiratory
Acid base disorders - acidosis alkalosis metabolic respiratoryChetan Ganteppanavar
 
Arterial Blood Gases Interpretation
Arterial Blood Gases InterpretationArterial Blood Gases Interpretation
Arterial Blood Gases InterpretationDang Thanh Tuan
 

What's hot (20)

Abg by dr girish
Abg by dr girishAbg by dr girish
Abg by dr girish
 
ABG (Emergency Medicine)
ABG (Emergency Medicine)ABG (Emergency Medicine)
ABG (Emergency Medicine)
 
Abg Interpretation
Abg InterpretationAbg Interpretation
Abg Interpretation
 
SIMPLE AND SYSTEMATIC APPROACH TO Acid base disorders
SIMPLE AND SYSTEMATIC APPROACH TO Acid base disorders  SIMPLE AND SYSTEMATIC APPROACH TO Acid base disorders
SIMPLE AND SYSTEMATIC APPROACH TO Acid base disorders
 
ABG INTERPRETATION.pptx
ABG INTERPRETATION.pptxABG INTERPRETATION.pptx
ABG INTERPRETATION.pptx
 
Blood gas analysis case scenarios
Blood gas analysis case scenariosBlood gas analysis case scenarios
Blood gas analysis case scenarios
 
Basics In Arterial Blood Gas Interpretation
Basics In Arterial Blood Gas InterpretationBasics In Arterial Blood Gas Interpretation
Basics In Arterial Blood Gas Interpretation
 
Abg.2 Arterial blood gas analysis and example interpretation
Abg.2 Arterial blood gas analysis and example interpretationAbg.2 Arterial blood gas analysis and example interpretation
Abg.2 Arterial blood gas analysis and example interpretation
 
Arterial Blood Gases Talk
Arterial Blood Gases TalkArterial Blood Gases Talk
Arterial Blood Gases Talk
 
Acid base balance
Acid base balanceAcid base balance
Acid base balance
 
ABG , ARTERIAL BLOOD GAS
ABG , ARTERIAL BLOOD GAS ABG , ARTERIAL BLOOD GAS
ABG , ARTERIAL BLOOD GAS
 
Mcq exam 2011
Mcq exam 2011Mcq exam 2011
Mcq exam 2011
 
Interpreting Blood Gases, Practical and easy approach
Interpreting Blood Gases, Practical and easy approachInterpreting Blood Gases, Practical and easy approach
Interpreting Blood Gases, Practical and easy approach
 
Arterial Blood Gases Analysis
Arterial Blood Gases AnalysisArterial Blood Gases Analysis
Arterial Blood Gases Analysis
 
Community acquired pneumonia
Community acquired pneumoniaCommunity acquired pneumonia
Community acquired pneumonia
 
Arterial blood gas
Arterial blood gasArterial blood gas
Arterial blood gas
 
RT ARTERIAL BLOOD GAS .ppt
RT ARTERIAL BLOOD GAS .pptRT ARTERIAL BLOOD GAS .ppt
RT ARTERIAL BLOOD GAS .ppt
 
Acid base disorders - acidosis alkalosis metabolic respiratory
Acid base disorders -  acidosis alkalosis metabolic respiratoryAcid base disorders -  acidosis alkalosis metabolic respiratory
Acid base disorders - acidosis alkalosis metabolic respiratory
 
Acid Base Status
Acid Base StatusAcid Base Status
Acid Base Status
 
Arterial Blood Gases Interpretation
Arterial Blood Gases InterpretationArterial Blood Gases Interpretation
Arterial Blood Gases Interpretation
 

Viewers also liked

Arterial Blood Gas Analysis
 Arterial Blood Gas Analysis Arterial Blood Gas Analysis
Arterial Blood Gas Analysisintentdoc
 
Advanced Acid Base on the Pediatric Intensive Care Unit
Advanced Acid Base on the Pediatric Intensive Care UnitAdvanced Acid Base on the Pediatric Intensive Care Unit
Advanced Acid Base on the Pediatric Intensive Care UnitDavid Schmidt
 
Quantitative Acid Base Analysis Fencl Stewart Approach
Quantitative Acid Base Analysis Fencl Stewart ApproachQuantitative Acid Base Analysis Fencl Stewart Approach
Quantitative Acid Base Analysis Fencl Stewart Approachmcrohman
 
Normal Saline is not Normal? ; Chloride liberal vs. Chloride restrictive IV F...
Normal Saline is not Normal? ; Chloride liberal vs. Chloride restrictive IV F...Normal Saline is not Normal? ; Chloride liberal vs. Chloride restrictive IV F...
Normal Saline is not Normal? ; Chloride liberal vs. Chloride restrictive IV F...Wisit Cheungpasitporn
 
Stewart approach in acid base balance
Stewart approach in acid base balanceStewart approach in acid base balance
Stewart approach in acid base balanceDr Iyan Darmawan
 
11 acid base regulation
11   acid base regulation11   acid base regulation
11 acid base regulationMUBOSScz
 
Non-anion gap Metabolic Acidosis (NAGMA)
Non-anion gap Metabolic Acidosis (NAGMA)Non-anion gap Metabolic Acidosis (NAGMA)
Non-anion gap Metabolic Acidosis (NAGMA)Joel Topf
 

Viewers also liked (10)

Arterial Blood Gas Analysis
 Arterial Blood Gas Analysis Arterial Blood Gas Analysis
Arterial Blood Gas Analysis
 
Advanced Acid Base on the Pediatric Intensive Care Unit
Advanced Acid Base on the Pediatric Intensive Care UnitAdvanced Acid Base on the Pediatric Intensive Care Unit
Advanced Acid Base on the Pediatric Intensive Care Unit
 
Quantitative Acid Base Analysis Fencl Stewart Approach
Quantitative Acid Base Analysis Fencl Stewart ApproachQuantitative Acid Base Analysis Fencl Stewart Approach
Quantitative Acid Base Analysis Fencl Stewart Approach
 
Normal Saline is not Normal? ; Chloride liberal vs. Chloride restrictive IV F...
Normal Saline is not Normal? ; Chloride liberal vs. Chloride restrictive IV F...Normal Saline is not Normal? ; Chloride liberal vs. Chloride restrictive IV F...
Normal Saline is not Normal? ; Chloride liberal vs. Chloride restrictive IV F...
 
10. abg analysis
10. abg analysis10. abg analysis
10. abg analysis
 
Stewart approach in acid base balance
Stewart approach in acid base balanceStewart approach in acid base balance
Stewart approach in acid base balance
 
11 acid base regulation
11   acid base regulation11   acid base regulation
11 acid base regulation
 
Non-anion gap Metabolic Acidosis (NAGMA)
Non-anion gap Metabolic Acidosis (NAGMA)Non-anion gap Metabolic Acidosis (NAGMA)
Non-anion gap Metabolic Acidosis (NAGMA)
 
Abg short seminar
Abg short seminarAbg short seminar
Abg short seminar
 
Metabolic acidosis
Metabolic acidosisMetabolic acidosis
Metabolic acidosis
 

Similar to Presentation1

abg objetives.pptx
abg objetives.pptxabg objetives.pptx
abg objetives.pptxjavier
 
abg-121230031055-phpapp02.pdf
abg-121230031055-phpapp02.pdfabg-121230031055-phpapp02.pdf
abg-121230031055-phpapp02.pdfDivyanshJoshi39
 
Arterial blood gas analysis in clinical practice (2)
Arterial blood gas analysis in clinical practice (2)Arterial blood gas analysis in clinical practice (2)
Arterial blood gas analysis in clinical practice (2)Mohit Aggarwal
 
arterial blood gas analysis
 arterial blood gas analysis arterial blood gas analysis
arterial blood gas analysishanaa
 
Arterial blood gas analysis 1
Arterial blood gas analysis 1Arterial blood gas analysis 1
Arterial blood gas analysis 1Ajay Kurian
 
Acid Base Disorders
Acid Base DisordersAcid Base Disorders
Acid Base DisordersDrSnehaDas
 
Acid Base disorder Concept.pptx
Acid Base disorder  Concept.pptxAcid Base disorder  Concept.pptx
Acid Base disorder Concept.pptximrulsujon1
 
Acid Base disorder Concept.pptx
Acid Base disorder  Concept.pptxAcid Base disorder  Concept.pptx
Acid Base disorder Concept.pptxImrul Sujon
 
Acid base and ABG interpretation in ICU
Acid base and ABG interpretation in  ICUAcid base and ABG interpretation in  ICU
Acid base and ABG interpretation in ICUAnwar Yusr
 
Arterial blood gas analysis
Arterial blood gas analysisArterial blood gas analysis
Arterial blood gas analysisKrishna Yadarala
 
ABG interpret in critical care 16-1-2024
ABG interpret in critical care 16-1-2024ABG interpret in critical care 16-1-2024
ABG interpret in critical care 16-1-2024Anwar Yusr
 
acid base and electrolye disorder in ICU.ppt
acid base and electrolye disorder in ICU.pptacid base and electrolye disorder in ICU.ppt
acid base and electrolye disorder in ICU.pptMisganawMengie
 
ARTERIAL BLOOD GASES INTERPRETATION
ARTERIAL BLOOD GASES INTERPRETATIONARTERIAL BLOOD GASES INTERPRETATION
ARTERIAL BLOOD GASES INTERPRETATIONDr.RMLIMS lucknow
 

Similar to Presentation1 (20)

ABG Interpretation
ABG InterpretationABG Interpretation
ABG Interpretation
 
Abg
AbgAbg
Abg
 
abg objetives.pptx
abg objetives.pptxabg objetives.pptx
abg objetives.pptx
 
abg-121230031055-phpapp02.pdf
abg-121230031055-phpapp02.pdfabg-121230031055-phpapp02.pdf
abg-121230031055-phpapp02.pdf
 
The biochemical aspect of pH imbalance
The biochemical aspect of pH imbalanceThe biochemical aspect of pH imbalance
The biochemical aspect of pH imbalance
 
Arterial blood gas analysis in clinical practice (2)
Arterial blood gas analysis in clinical practice (2)Arterial blood gas analysis in clinical practice (2)
Arterial blood gas analysis in clinical practice (2)
 
Abc of abg richa j
Abc of abg richa jAbc of abg richa j
Abc of abg richa j
 
arterial blood gas analysis
 arterial blood gas analysis arterial blood gas analysis
arterial blood gas analysis
 
ABG Analysis & Interpretation
ABG Analysis & InterpretationABG Analysis & Interpretation
ABG Analysis & Interpretation
 
Arterial blood gas analysis 1
Arterial blood gas analysis 1Arterial blood gas analysis 1
Arterial blood gas analysis 1
 
Acid Base Disorders
Acid Base DisordersAcid Base Disorders
Acid Base Disorders
 
ABG APPROACH
ABG APPROACHABG APPROACH
ABG APPROACH
 
abg FINAL.pptx
abg FINAL.pptxabg FINAL.pptx
abg FINAL.pptx
 
Acid Base disorder Concept.pptx
Acid Base disorder  Concept.pptxAcid Base disorder  Concept.pptx
Acid Base disorder Concept.pptx
 
Acid Base disorder Concept.pptx
Acid Base disorder  Concept.pptxAcid Base disorder  Concept.pptx
Acid Base disorder Concept.pptx
 
Acid base and ABG interpretation in ICU
Acid base and ABG interpretation in  ICUAcid base and ABG interpretation in  ICU
Acid base and ABG interpretation in ICU
 
Arterial blood gas analysis
Arterial blood gas analysisArterial blood gas analysis
Arterial blood gas analysis
 
ABG interpret in critical care 16-1-2024
ABG interpret in critical care 16-1-2024ABG interpret in critical care 16-1-2024
ABG interpret in critical care 16-1-2024
 
acid base and electrolye disorder in ICU.ppt
acid base and electrolye disorder in ICU.pptacid base and electrolye disorder in ICU.ppt
acid base and electrolye disorder in ICU.ppt
 
ARTERIAL BLOOD GASES INTERPRETATION
ARTERIAL BLOOD GASES INTERPRETATIONARTERIAL BLOOD GASES INTERPRETATION
ARTERIAL BLOOD GASES INTERPRETATION
 

Presentation1

  • 1. Arterial Blood Gas Interpretation PRESENTER- Dr. Shankerdeep Sondhi Resident IIInd Yr Department of Medicine
  • 2. OBJECTIVES ABG Sampling Interpretation of ABG Case Scenarios
  • 3. ABG – Procedure and Precautions  Site- (Ideally) Radial Artery Brachial Artery Femoral Artery  Ideally - Pre-heparinised ABG syringes - Syringe should be FLUSHED with 0.5ml of 1:1000 Heparin solution and emptied. DO NOT LEAVE EXCESSIVE HEPARIN IN THE SYRINGE HEPARIN DILUTIONAL HCO3 EFFECT PCO2 Only small 0.5ml Heparin for flushing and discard it Syringes must have > 50% blood.
  • 4. Ensure No Air Bubbles. Syringe must be sealed immediately after withdrawing sample. ◦ Contact with AIR BUBBLES Air bubble = PO2 150 mm Hg , PCO2 0 mm Hg Air Bubble + Blood = PO2 PCO2  ABG Syringe must be transported at the earliest to the laboratory for EARLY analysis via COLD CHAIN
  • 5. ABG ELECTRODES A. pH (Sanz Electrode)  Measures H+ ion concentration of sample against a known pH in a reference electrode, hence potential difference. Calibration with solutions of known pH (6.384 to 7.384) B. P CO2 (Severinghaus Electrode)  CO2 reacts with solution to produce H+ higher C02- more H+  higher P CO2 measured C. P 02 (Clark Electrode)  02 diffuses across membrane producing an electrical current measured as P 02.
  • 7. Acid Base Homeostasis 1. Plasma Acid Homeomostasis: Chemical buffering ◦ H+ influenced by  Rate of endogenous production  Rate of excretion  Buffering capacity of body ◦ Buffers effective at physiologic pH are  Hemoglobin  Phosphate  Proteins  Bicarbonate
  • 8. 2. Alveolar Ventilation: pCO2 action is immediate. Stimulation of respiratory center washes off the excess CO2 increasing pH, and vice versa 3. Excretion: Liver: uses HCO3– to make urea which further prevents accumulation of ammonia and traps H+ in renal distal tubule Kidney: regulate plasma [HCO3–] through three main processes: • reabsorption of filtered HCO3– • formation of titratable acid • excretion of NH4+ in the urine. Proximal tubule reclaims 85% filtered HCO3– Distal tubule reclaims 15%, and excretes H+ 40–60 mmol/d of protons is excreted to maintain balance
  • 9. Acid Base Balance  H+ ion concentration in the body is precisely regulated  The body understands the importance of H+ and hence devised DEFENCES against any change in its concentration- BICARBONATE RESPIRATORY RENAL BUFFER REGULATION REGULATION SYSTEM Acts in few Acts in hours to Acts in few minutes days seconds
  • 10. BUFFER SYSTEM  1st line of defence in pH regulation.  Determine capacity of ECF to transport acids from site of production to site of excretion without undue change in pH.  They resist change in pH on addition of acid/alkali in media wherever they are.
  • 11. PLASMA RBC BUFFERS BUFFERS  NaHCO3/H2CO  KHCO3/H2CO3 3  K2HPO4/KH2P4  Na2HPO4/  KHb/HHb NaH2PO4  Na-Pr/H-Pr
  • 12. BICARBONATE BUFFER  NAHCO3/H2CO3 = 20/1 = Alkali reserve.  conversion of strong & non volatile acid in ECF, in weak & volatile acid at the expense of NaHCO3 component of buffer.  H2CO3 thus formed is eliminated by lung as CO2.Directly linked with respiration & healthy lungs essential for its function.  High concentration in blood,so very good physiological buffer.  Weak chemical buffer.
  • 13.  Bicarbonate Buffer System CO2 + H2O carbonic anhydrase H2CO3 H+ + HCO3- In Acidosis - Acid = H+ H+ + HCO3 H2CO3 CO2 + H2O In Alkalosis - Alkali + Weak Acid = H2CO3 CO2 + H20 H2CO3 HCO3- + H+ + ALKALI
  • 14. PHOSPHATE BUFFER  Na2HPO4/NaH2PO4 = AlkPO4/AcidPO4 = 4/1.  NaH2PO4 excreted by kidneys. Directly linked with kidneys & healthy kidneys necessary for its functioning.  Concentration in blood is low so not good physiological buffer.  Very good chemical buffer as pKa aprroches pH.
  • 15. PROTEIN BUFFER  Na+Pr- / H+Pr- = salt/acid.  Proteins can act as a base In acidic medium nd as an acid in basic medium..with COOH& NH2 group.  Buffering capacity of plasma proteins is much less than Hb.
  • 16.  Respiratory Regulation of Acid Base Balance- ALVEOLAR H+ VENTILATION PaCO2 ALVEOLAR H+ VENTILATION PaCO2
  • 17.  Renal Regulation of Acid Base Balance Kidneys control the acid-base balance by excreting either an acidic or a basic urine, This is achieved in the following ways- Reabsorption Secretion of H+ of HCO3 ions in tubules in blood and excretion PCO2 K+ in ECF Aldosterone H+ ion •Proximal Convulated Tubules (85%) •Thick Ascending Limb of ECF Volume Loop of Henle (10%) Angiotensin II •Distal Convulated Tubule •Collecting Tubules(5%)
  • 18. Definitions and Terminology  ACIDOSIS – presence of a process which tends to pH by virtue of gain of H + or loss of HCO3-  ALKALOSIS – presence of a process which tends to pH by virtue of loss of H+ or gain of HCO3- If these changes, change pH, suffix ‘emia’ is added  ACIDEMIA – reduction in arterial pH (pH<7.35)  ALKALEMIA – increase in arterial pH (pH>7.45)
  • 19.  Simple Acid Base Disorder/ Primary Acid Base disorder – a single primary process of acidosis or alkalosis due to an initial change in PCO2 and HCO3.  Compensation - The normal response of the respiratory system or kidneys to change in pH induced by a primary acid-base disorder  The Compensatory responses to a primary Acid Base disturbance are never enough to correct the change in pH , they only act to reduce the severity.  Mixed Acid Base Disorder – Presence of more than one acid base disorder simultaneously .
  • 20. Normogram (for simple acid-base ds)
  • 21. Characteristics of Primary ACID BASE Disorders PRIMARY PRIMARY RESPONSES COMPENSATORY DISORDER H+ ion pH Primary RESPONSES Conc. Defect Metabolic PCO2 Acidosis H+ pH HCO3 Alveolar Hyperventilation Metabolic PCO2 Alkalosis H+ pH HCO3 Alveolar Hypoventilation Respiratory Acidosis H+ pH PCO2 HCO3 Respiratory Alkalosis H+ pH PCO2 HCO3
  • 22. Compensation Metabolic Disorders – Compensation in these disorders leads to a change in PCO2 METABOLIC ACIDOSIS • For every 1mmol/l in HCO3 the PCO2 falls by 1.25 mm Hg METABOLIC ALKALOSIS • For every 1mol/l in HCO3 the PCO2 by 0.75 mm Hg
  • 23. In Respiratory Disorders PCO2 Kidney HCO3 Reabsorption Compensation begins to appear in 6 – 12 hrs and is fully developed only after a few days. 1.ACUTE Before the onset of compensation Resp. acidosis – 1mmHg in PCO2 HCO3 by 0.1meq/l Resp. alkalosis – 1mmHg in PCO2 HCO3 by 0.2 meq/l 2.CHRONIC (>24 hrs) After compensation is fully developed Resp. acidosis – 1mmHg in PCO2 HCO3 by 0.4meq/l Resp. alkalosis – 1mmHg in PCO2 HCO3 by 0.4meq/l
  • 24. Body’s physiologic response to Primary disorder in order to bring pH towards NORMAL limit Full compensation Partial compensation No compensation…. (uncompensated) BUT never overshoots, If a overshoot pH is there, Take it granted it is a MIXED disorder
  • 25. STEP WISE APPROACH to Interpretation Of ABG reports
  • 26. Normal Values ANALYTE Normal Value Units pH 7.35 - 7.45 PCO2 35 – 45 mm Hg PO2 80 – 100 mm Hg` [HCO3] 22 – 26 meq/L SaO2 95-100 % Anion Gap 10 + 2 meq/L ∆HCO3 +2 to -2 meq/L
  • 27. STEP 0 • Is this ABG Authentic? STEP 1 • ACIDEMIA or ALKALEMIA? • RESPIRATORY or METABOLIC? STEP 2 STEP 3 • Is COMPENSATION adequate? STEP 4 • If METABOLIC – ANION GAP? • If High gap Metabolic Acidosis– STEP 5 GAP GAP?
  • 28. Step 0 – Authentic or Not?  Verify that the ABG values are internally accurate. ◦ The accuracy of the values can be established by confirming that they satisfy a simplified form of the Henderson-Hasselbalch equation: [H+](nmol/L) = 24 × pCO2(mm Hg) ∕ [HCO3-] (mEq/L) ◦ [H+] = 10 exp(-pH). Within the pH range of 7.26 to 7.45 [H+] in nmol/L = 80 − the decimal of the pH (e.g., for pH = 7.25, [H+] = 80 − 25 = 55
  • 29. Example: using Kassirer- Bleich equation [H+](nmol/L) = 24 × pCO2(mm Hg) ∕ [HCO3-] (mEq/L) ABG: pH = 7.25, pCO2 = 30, HCO3- = 22 30 55 = 24 x 22 55 ≠ 32 LAB ERROR
  • 30. STEP 1 ACIDEMIA OR ALKALEMIA?  Look at pH <7.35 - acidemia >7.45 – alkalemia  RULE – An acid base abnormality is present even if either the pH or PCO2 are Normal.
  • 31. STEP 2 RESPIRATORY or METABOLIC? IS PRIMARY DISTURBANCE RESPIRATORY OR METABOLIC? pH HCO3 or pH HCO3 METABOLIC pH PCO2 or pH PCO2 RESPIRATORY RULE- If either the pH or PCO2 is Normal, there is a mixed metabolic and respiratory acid base disorder.
  • 32. Step 3 Is Compensation Adequate?
  • 33. Disorder Prediction of Compensation pH HCO3– PaCO2 Metabolic PaCO2 will 1.25 mmHg per mmol/L in acidosis [HCO3-] Low Low Low Metabolic PaCO2 will 0.75 mmHg per mmol/L in High High High alkalosis [HCO3-]
  • 34. Disorder Prediction of Compensation pH HCO3– PaCO2 Respiratory High Low Low alkalosis Acute [HCO3-] will 0.2 mmol/L per mmHg in PaCO2 Chronic [HCO3-] will 0.4 mmol/L per mmHg in PaCO2 Respiratory Low High High acidosis Acute [HCO3-] will 0.1 mmol/L per mmHg in PaCO2 Chronic [HCO3-] will 0.4 mmol/L per mmHg in PaCO2
  • 35. STEP 0 • Is this ABG Authentic? STEP 1 • ACIDEMIA or ALKALEMIA? • RESPIRATORY or METABOLIC? STEP 2 STEP 3 • If Respiratory – ACUTE or CHRONIC? STEP 4 • Is COMPENSATION adequate? STEP 4 • If METABOLIC – ANION GAP? • If High gap Metabolic Acidosis– STEP 6 GAP GAP?
  • 36. Electrochemical Balance in Blood 100% UC UA 90% 80% HCO3 Na Sulfate 70% Phosphate 60% Mg- OA Cl 50% K - Proteins 40% Ca-HCO3 30% Na- Cl 20% 10% 0% CATIONS ANIONS
  • 37. Anion Gap AG based on principle of electroneutrality:  Total Serum Cations = Total Serum Anions  M cations + U cations = M anions + U anions  Na + (K + Ca + Mg) = HCO3 + Cl + (PO4 + SO4 + Protein + Organic Acids)  Na + UC = HCO3 + Cl + UA  But in Blood there is a relative abundance of Anions, hence Anions > Cations  Na – (HCO3 + Cl) = UA – UC  Na – (HCO3 + Cl) = Anion Gap
  • 38. METABOLIC ACIDOSIS- STEP 4 ANION GAP? IN METABOLIC ACIDOSIS WHAT IS THE ANION GAP? ANION GAP(AG) = Na – (HCO3 + Cl) Normal Value = 10 + 2 Adjusted Anion Gap = Observed AG +2.5(4.5- S.Albumin) 50% in S. Albumin 75% in Anion Gap !!! High Anion Gap Metabolic Acidosis Metabolic Acidosis Normal Anion Gap Acidosis
  • 39. STEP 5 CO EXISTANT METABOLIC DISORDER – “Gap Gap‖? C/O HGAG METABOLIC ACIDOSIS,ANOTHER DISORDER?  ∆ Anion Gap = Measured AG – Normal AG Measured AG – 12 ∆ HCO3 = Normal HCO3 – Measured HCO3 24 – Measured HCO3 Ideally, ∆Anion Gap = ∆HCO3 For each 1 meq/L increase in AG, HCO3 will fall by 1 meq/L ∆AG/ HCO3- = 1  Pure High AG Met Acidosis AG/ HCO3- > 1  Assoc Metabolic Alkalosis AG/ HCO3- < 1  Assoc N AG Met Acidosis
  • 40. CLINICAL CASE SCENARIO
  • 41. Case Scenario A patient with a severe postoperative ileus requires the insertion of a nasogastric (NG) tube for decompression. After several days on the floor, he develops a line infection and is moved to the ICU once he becomes pressor dependent. The patient's ABG reveals a pH = 7.44, pCO2 = 12, and [HCO3-] = 8. The [Na+] = 145 with [Cl-] = 102.
  • 42. ABG: pH = 7.44, pCO2 = 12, [HCO3-] = 8, [Na+] = 145 with [Cl-] = 102 ◦ With knowledge of the common clinical scenarios leading to acid-base disturbances, this patient is at risk for developing a metabolic alkalosis from NG suction and a metabolic acidosis and respiratory alkalosis from sepsis. ◦ Step 1. [H+] = 80 − 44 = 36. Does 36 = 24 × 12 / 8? It does. ◦ Step 2. The patient is mildly alkalemic, which can be explained by the low pCO2 but not by the low [HCO3-], suggesting that a respiratory alkalosis may be the primary derangement.
  • 43. ABG: pH = 7.44, pCO2 = 12, [HCO3-] = 8, [Na+] = 145 with [Cl-] = 102 ◦ Step 3  a. The drop in [HCO3-] might be an appropriate compensation for a chronic respiratory alkalosis ([HCO3-] is 0.4 mmol/L per mmHg in PaCO2) However, 24 − [(40 − 12) × 0.4] = 12.8, which is not close to the observed [HCO3-] of 8. A mixed disorder is implied.  b. AG = 145 − 102 − 8 = 35. There is an elevated AG, suggests a concomitant metabolic acidosis.  c. Delta anion gap=35 − 10 = 25.  d. Delta bicarbonate= 24 − 8 =16.  Ratio of above two is more than 1 suggestive of associated metabolic alkalosis. This has proven to be a triple acid-base disorder with an elevated anion gap acidosis, a metabolic alkalosis, and a respiratory alkalosis, as was alluded to in Step 1
  • 44. Metabolic Acidosis  Metabolic acidosis can occur because of ◦ increase in endogenous acid production (lactate and ketoacids) ◦ loss of bicarbonate (as in diarrhea) ◦ accumulation of endogenous acids (as in renal failure).  Effects on the respiratory, cardiac, and nervous systems. ◦ Kussmaul respiration ◦ Intrinsic cardiac contractility may be depressed, but inotropic function can be normal because of catecholamine release. ◦ Both peripheral arterial vasodilation and central venoconstriction can be present ◦ The decrease in central and pulmonary vascular compliance predisposes to pulmonary edema with even minimal volume overload. ◦ Central nervous system function is depressed, with headache, lethargy, stupor, and, in some cases, even coma.
  • 45. Metabolic Acidosis: Essentials of Diagnosis  Decreased HCO3– with acidemia.  Classified into high anion gap acidosis and normal anion gap (hyperchloremic) acidosis.  The high anion gap acidoses are seen in lactic acidosis, ketoacidosis, renal failure or toxins.  Normal anion gap acidosis is mainly caused by gastrointestinal HCO3– loss or RTA.  Urinary anion gap may help distinguish between these causes.
  • 46. Causes of High-Anion-Gap Metabolic Acidosis 1. Lactic acidosis  Poor tissue perfusion (type A)  Aerobic disorders (type B) 2. Ketoacidosis  Diabetic  Alcoholic  Starvation 3. Toxins  Ethylene glycol  Methanol  Salicylates  Propylene glycol: as vehicle of medications  Pyroglutamic acid: acetaminophen toxicity 4. Renal failure (acute and chronic)
  • 47. Ketacidosis – Diabetic Ketoacidosis (DKA):  DKA is caused by increased fatty acid metabolism and the accumulation of ketoacids (acetoacetate and -hydroxybutyrate).  DKA usually occurs in insulin-dependent diabetes mellitus in association with  cessation of insulin or  an intercurrent illness, such as an infection, gastroenteritis, pancreatitis, or myocardial infarction, which increases insulin requirements temporarily and acutely.  The accumulation of ketoacids accounts for the increment in the Anion Gap and is accompanied most often by hyperglycemia [glucose > 17 mmol/L (300 mg/dL)].
  • 48. Glucose,a mmol/L (mg/dL) 13.9–33.3 (250–600) Sodium, meq/L 125–135 Potassiuma Normal to ↑ Magnesiuma Normal ( plasma levels may be normal or high at presentation, total-body stores are usually depleted) Chloridea Normal Phosphatea ↓ Creatinine Slightly ↑ Osmolality (mOsm/mL) 300–320 Plasma ketonesa ++++ Serum bicarbonate,a meq/L <15 meq/L Arterial pH 6.8–7.3 Arterial PCO2,a mmHg 20–30 Anion gapa[Na - (Cl + HCO3)] ↑
  • 49. Management  Confirm diagnosis (plasma glucose, positive serum ketones, metabolic acidosis).  Admit to hospital; intensive-care setting may be necessary for frequent monitoring or if pH < 7.00 or unconscious.  Assess:  Serum electrolytes (K+, Na+, Mg2+, Cl-, bicarbonate, phosphate)  Acid-base status—pH, HCO3-, PCO2, b-hydroxybutyrate  Renal function (creatinine, urine output)  Replace fluids:  2–3 L of 0.9% saline over first 1–3 h (10–15 mL/kg per hour);  subsequently, 0.45% saline at 150–300 mL/h;  change to 5% glucose and 0.45% saline at 100–200 mL/h when plasma glucose reaches 250 mg/dL (14 mmol/L).  Administer short-acting insulin:  IV (0.1 units/kg) or IM (0.3 units/kg), then 0.1 units/kg per hour by continuous IV infusion;  increase 2- to 3-fold if no response by 2–4 h.  If initial serum potassium is < 3.3 mmol/L (3.3 meq/L), do not administer insulin until the potassium is corrected to > 3.3 mmol/L
  • 50. Management  Assess patient: What precipitated the episode (noncompliance, infection, trauma, infarction, cocaine)? Initiate appropriate workup for precipitating event (cultures, CXR, ECG).  Measure  capillary glucose every 1–2 h;  electrolytes (especially K+, bicarbonate, phosphate) and anion gap every 4 h for first 24 h.  Monitor blood pressure, pulse, respirations, mental status, fluid intake and output every 1–4 h.  Replace K+:  10 meq/h when plasma K+ < 5.5 meq/L, ECG normal, urine flow and normal creatinine documented;  administer 40–80 meq/h when plasma K+ < 3.5 meq/L or if bicarbonate is given.  Continue above until patient is stable, glucose goal is 150– 250 mg/dL, and acidosis is resolved. Insulin infusion may be decreased to 0.05–0.1 units/kg per hour.  Administer intermediate or long-acting insulin as soon as patient is eating. Allow for overlap in insulin infusion and
  • 51.  since insulin prevents production of ketones, bicarbonate therapy is rarely needed except with extreme acidemia (pH < 7.1), and then in only limited amounts.  Patients with DKA are typically volume depleted and require fluid resuscitation with isotonic saline.  Volume overexpansion is not uncommon after IV fluid administration, and contributes to the development of a hyperchloremic acidosis during treatment of DKA because volume expansion increases urinary ketoacid anion excretion (loss of potential bicarbonate).  The mainstay for treatment of this condition is IV regular insulin
  • 52. Drug, Toxin-Induced Acidosis  Plasma osmolality is calculated according to the following expression: Posm = 2Na+ + Glu + BUN (all in mmol/L) Posm = 2Na+ + Glu/18 + BUN/2.8 (milligrams per deciliter)  The calculated and determined osmolality should be within 10–15 mmol/kg H2O  When the measured osmolality exceeds the calculated osmolality by >15–20 mmol/kg H2O, either prevails.  Either the serum sodium is spuriously low, as with hyperlipidemia or hyperproteinemia (pseudohyponatremia)  Osmolytes other than sodium salts, glucose, or urea have accumulated in plasma. Examples include mannitol, radiocontrast media, isopropyl alcohol, ethylene glycol, propylene
  • 53. In this situation, the difference between the calculated osmolality and the measured osmolality (osmolar gap) is proportional to the concentration of the unmeasured solute.  Alcohols: With an appropriate clinical history and index of suspicion, identification of an osmolar gap is helpful in identifying the presence of poison-associated AG acidosis. Three alcohols may cause fatal intoxications: ethylene glycol, methanol, and isopropyl alcohol  Salicylates: Salicylate intoxication in adults usually causes respiratory alkalosis or a mixture of high-AG metabolic acidosis and respiratory alkalosis. Only a portion of the AG is due to salicylates. Lactic acid production is also often increased
  • 54. Renal failure Acidosis  The hyperchloremic acidosis of moderate renal insufficiency is eventually converted to the high-AG acidosis of advanced renal failure.  At GFRs below 20 mL/min, the inability to excrete H+ with retention of acid anions such as PO43– and SO42– results in an increased anion gap acidosis  [HCO3–] rarely falls to <15 mmol/L, and the AG is rarely >20 mmol/L, indicating that the acid retained in chronic renal disease is buffered by alkaline salts from bone.  Results in significant loss of bone mass due to reduction in bone calcium carbonate and increases urinary calcium excretion.
  • 55. Treatment of high anion gap acidosis  Treatment is aimed at the underlying disorder, such as insulin and fluid therapy for diabetes and appropriate volume resuscitation to restore tissue perfusion. The metabolism of lactate will produce HCO3– and increase pH.  Controversy regarding administration of large amounts of HCO3–  may have deleterious effects, including hypernatremia and hyperosmolality.  Intracellular pH may decrease because administered HCO3– is converted to CO2, which easily diffuses into cells, combines with water to create additional hydrogen ions and worsening of intracellular acidosis and this could impair cellular function  Alkali administration is known to stimulate phosphofructokinase activity, thus exacerbating lactic acidosis via enhanced lactate production. Ketogenesis is also augmented by alkali therapy.
  • 56. Treatment of high anion gap acidosis….. ◦ In salicylate intoxication, alkali therapy must be started unless blood pH is already alkalinized by respiratory alkalosis, since the increment in pH converts salicylate to more impermeable salicylic acid and thus prevents central nervous system damage. ◦ In DKA, alkali must be administered in extreme alkalemia (pH<7.1) ◦ In alcoholic ketoacidosis, thiamine should be given together with glucose to avoid the development of Wernicke's encephalopathy.  The amount of HCO3– deficit can be calculated as follows: Amount of HCO3– deficit = 0.5 X body weight X (24 - HCO3–) ◦ Half of the calculated deficit should be administered within the first 3–4 hours to avoid overcorrection and volume overload.  In methanol intoxication ◦ ethanol has been used as a competitive substrate for alcohol dehydrogenase, which metabolizes methanol to formaldehyde ◦ or through direct inhibition of alcohol dehydrogenase by fomepizole
  • 57. Hyperchloremic (Nongap) Metabolic Acidoses: causes I. Gastrointestinal bicarbonate loss  Diarrhea  External pancreatic or small-bowel drainage  Ureterosigmoidostomy, jejunal loop, ileal loop  Drugs - Calcium chloride , Magnesium sulfate (diarrhea), Cholestyramine (bile acid diarrhea) II. Renal acidosis  Hypokalemia  Proximal RTA (type 2)  Distal (classic) RTA (type 1)  Hyperkalemia  Generalized distal nephron dysfunction (type 4 RTA)  a. Mineralocorticoid deficiency  b. Mineralocorticoid resistance (autosomal dominant PHA I)  c. Voltage defect (autosomal dominant PHA I and PHA II)  d. Tubulointerstitial disease
  • 58. III. Drug-induced hyperkalemia (with renal insufficiency) ◦ Potassium-sparing diuretics (amiloride, triamterene, spironolactone) ◦ Trimethoprim ◦ Pentamidine ◦ ACE-Is and ARBs ◦ Nonsteroidal anti-inflammatory drugs ◦ Cyclosporine and tacrolimus IV. Other ◦ Acid loads (ammonium chloride, hyperalimentation) ◦ Loss of potential bicarbonate: ketosis with ketone excretion ◦ Expansion /dilutional acidosis (rapid saline administration) ◦ Hippurate ◦ Cation exchange resins
  • 59. Approach: Urinary Anion Gap  Increased renal NH4+Cl– excretion to enhance H+ removal is a normal physiologic response to metabolic acidosis. NH3 reacts with H+ to form NH4+, which is accompanied by the anion Cl– for excretion.  Urinary anion gap from a random urine sample ([Na++ K+]– Cl–) reflects the ability of the kidney to excrete NH4Cl as in the following equation: Na+ + K+ + NH4+ = Cl– + 80 urinary anion gap is equal to (80 – NH4+)  Gastrointestinal HCO3– loss (diarrhea), the renal acidification ability remains normal and NH4Cl excretion increases in response to the acidosis. urinary anion gap is negative (eg, –30 mEq/L).  Distal RTA, the urinary anion gap is positive (eg, +25 mEq/L), since the basic lesion in the disorder is the inability of the kidney to excrete H+ and thus the inability to increase NH4Cl excretion.  Proximal (type II) RTA, the kidney has defective HCO3– reabsorption, leading to increased HCO3– excretion rather than decreased NH4Cl excretion. Thus, the urinary anion gap is negative
  • 60. Urinary pH may not as readily differentiate between the two causes because volume depletion or potassium depletion, which can accompany diarrhea (and surreptitious laxative abuse) may impair renal acidification.  Thus, when volume depletion is present, the urinary anion gap is a better measurement of ability to acidify the urine than urinary pH.  When large amounts of other anions are present in the urine, the urinary anion gap may not be reliable. In such a situation, NH4+ excretion can be estimated using the urinary osmolar gap.  NH4+ excretion (mmol/L) = 0.5 x Urinary osmolar gap = 0.5 [U osm – 2(U Na++U K+) + U urea + U glucose]  where urinary (U) concentrations and osmolality are in millimoles per liter.
  • 61. Renal Serum Urinary Titratable Urinary Treatment Defect [K+] NH4+ Plus Acid Anion Minimal Gap Urine pH Gastrointestinal None ↓ < 5.5 ↑↑ Negative Na+, K+, and HCO3– loss HCO3– as required Renal tubular acidosis I. Classic distal Distal H+ ↓ > 5.5 ↓ Positive NaHCO3 (1–3 secretion mEq/kg/d) II. Proximal Proximal ↓ < 5.5 Normal Negative NaHCO3 or secretion HCO3– KHCO3 (10–15 abspn mEq/kg/d), thiazide IV. Distal Na+ ↑ < 5.5 ↓ Positive Fludrocortisone Hyporeninemic reabsorption (0.1–0.5 mg/d), + dietary K+ rstrn, hypoaldosteroni K secretion, and H+ furosemide (40– sm secretion 160 mg/d), NaHCO3 (1–3 mEq/kg/d)
  • 62. Treatment of normal Anion gap Acidosis  Gastrointestinal: Correction of the contracted ECFV with NaCl and repair of K+ deficits corrects the acid-base disorder, and chloride deficiency.  RTA : administration of alkali (either as bicarbonate or citrate) to correct metabolic abnormalities and prevent nephrocalcinosis and renal failure. ◦ Proximal RTA : Large amounts of alkali (10–15 mEq/kg/d) may be required to treat proximal RTA because most of the alkali is excreted into the urine, which exacerbates hypokalemia. Thus, a mixture of sodium and potassium salts, such as K-Shohl solution, is preferred. The addition of thiazides may reduce the amount of alkali required, but hypokalemia may develop. ◦ Distal RTA : Correction of type 1 distal RTA requires a smaller amount of alkali (1–3 mEq/kg/d) and potassium supplementation as needed. ◦ Type IV RTA: dietary potassium restriction may be needed and potassium-retaining drugs should be withdrawn. Fludrocortisone may be effective in cases with hypoaldosteronism, but should be used with care, preferably in combination with loop diuretics. alkali supplementation (1–3 mEq/kg/d) may be required.
  • 63.
  • 64. Metabolic Alkalosis  Essentials of Diagnosis ◦ High HCO3– with alkalemia. ◦ Evaluate effective circulating volume by physical examination and check urinary chloride concentration. This will help differentiate saline-responsive metabolic alkalosis from saline-unresponsive alkalosis
  • 65. Metabolic Alkalosis  Occurs as a result of net gain of [HCO3–] or loss of nonvolatile acid (usually HCl by vomiting) from the extracellular fluid.  The disorder involves  a generative stage, in which the loss of acid usually causes alkalosis  maintenance stage, in which the kidneys fail to compensate by excreting HCO3–.  Classified based on "saline responsiveness" or urinary Cl–, which are markers for volume status  Saline-responsive metabolic alkalosis is a sign of extracellular volume contraction  Saline-unresponsive alkalosis implies a volume- expanded state
  • 66. Symptoms  With metabolic alkalosis, changes in central and peripheral nervous system function are similar to those of hypocalcemia : symptoms include  Mental confusion  Obtundation  Predisposition to seizures  Paresthesia  Muscular cramping  Tetany  Aggravation of arrhythmias  Hypoxemia in chronic obstructive pulmonary disease.  Related electrolyte abnormalities include  Hypokalemia- Weakness and hyporeflexia  hypophosphatemia.
  • 67. Classification and etiology I. Saline-Responsive (UCl < 10 mEq/d)  Excessive body bicarbonate content  Renal alkalosis  Diuretic therapy  Poorly reabsorbable anion therapy: carbenicillin, penicillin, sulfate, phosphate  Posthypercapnia  Gastrointestinal alkalosis  Loss of HCl from vomiting or nasogastric suction  Intestinal alkalosis: chloride diarrhea  Exogenous alkali  NaHCO3 (baking soda)  Sodium citrate, lactate, gluconate, acetate  Transfusions  Antacids  Normal body bicarbonate content  "Contraction alkalosis"
  • 68. II. Saline-Unresponsive (UCl > 10 mEq/d)  Excessive body bicarbonate content  Renal alkalosis  Normotensive  Bartter's syndrome (renal salt wasting and secondary hyperaldosteronism)  Severe potassium depletion  Refeeding alkalosis  Hypercalcemia and hypoparathyroidism  Hypertensive  Endogenous mineralocorticoids  Primary aldosteronism  Hyperreninism  Adrenal enzyme deficiency: 11- and 17- hydroxylase  Liddle's syndrome  Exogenous mineralocorticoids  Licorice
  • 69.
  • 70. Treatment  Mild alkalosis is generally well tolerated. Severe or symptomatic alkalosis (pH > 7.60) requires urgent treatment.  Saline-Responsive Metabolic Alkalosis ◦ Aimed at correction of extracellular volume deficit. ◦ Depending on the degree of hypovolemia, adequate amounts of 0.9% NaCl and KCl should be administered. ◦ Discontinuation of diuretics and administration of H2-blockers in patients whose alkalosis is due to nasogastric suction can be useful. ◦ If impaired pulmonary or cardiovascular status prohibits adequate volume repletion, acetazolamide, 250–500 mg intravenously every 4–6 hours, can be used.
  • 71. Treatment…..  One must be alert to the possible development of hypokalemia, since potassium depletion can be induced by forced kaliuresis via bicarbonaturia.  Administration of acid can be used as emergency therapy. HCl, 0.1 mol/L, is infused via a central vein (the solution is sclerosing). Dosage is calculated to decrease the HCO3– level by 50% over 2–4 hours, assuming an HCO3– volume of distribution (L) of 0.5% body weight (kg).  Patients with marked renal failure may require dialysis.  Saline-Unresponsive Metabolic Alkalosis  surgical removal of a mineralocorticoid-producing tumor  blockage of aldosterone effect with an ACE inhibitor or with spironolactone.  primary aldosteronism can be treated only with potassium repletion.
  • 72. Respiratory Acidosis Results from decreased alveolar ventilation and hypercapnia both due to pulmonary and non pulmonary disorders  Central  Neuromuscular ◦ Drugs ◦ Poliomyelitis (anesthetics, morphine, sedatives) ◦ Kyphoscoliosis ◦ Stroke ◦ Myasthenia ◦ Infection ◦ Muscular dystrophies  Airway  Miscellaneous ◦ Obstruction ◦ Obesity ◦ Asthma ◦ Hypoventilation  Parenchyma ◦ Permissive hypercapnia ◦ Emphysema ◦ Pneumoconiosis ◦ Bronchitis ◦ Adult respiratory distress syndrome ◦ Barotrauma
  • 73. Respiratory Acidosis  Acute respiratory failure  associated with severe acidosis and only a small increase in the plasma bicarbonate.  After 6–12 hours, the primary increase in PCO2 evokes a renal compensatory response to generate more HCO3– , which tends to ameliorate the respiratory acidosis. This takes several days to complete.  Chronic respiratory acidosis  seen in patients with underlying lung disease, such as chronic obstructive pulmonary disease.  Urinary excretion of acid in the form of NH4+ and Cl– ions results in the characteristic hypochloremia of chronic respiratory acidosis.  When chronic respiratory acidosis is corrected suddenly, especially in patients who receive mechanical ventilation, there is a 2- to 3-day lag in renal bicarbonate excretion, resulting in posthypercapnic metabolic alkalosis.
  • 74. Symptoms and Signs ◦ With acute onset, there is somnolence and confusion, and myoclonus with asterixis ◦ Coma from CO2 narcosis may ensue ◦ Severe hypercapnia increases cerebral blood flow and cerebrospinal fluid pressure. Signs of increased intracranial pressure (papilledema, pseudotumor cerebri) may be seen.  Laboratory Findings ◦ Arterial pH is low ◦ PCO2 is increased ◦ Serum HCO3– is elevated, but not enough to completely compensate for the hypercapnia. ◦ If the disorder is chronic, hypochloremia is seen.  Treatment ◦ In all forms of respiratory acidosis, treatment is directed at the underlying disorder to improve ventilation. ◦ Because opioid drug overdose is an important reversible cause of acute respiratory acidosis, naloxone, 0.04–2 mg intravenously is administered to all such patients if no obvious cause for respiratory depression is present. ◦ Mechanical ventilation for oxygenation till it restores back to normal maybe needed
  • 75. Respiratory Alkalosis(Hypocapnia)  Respiratory alkalosis, or hypocapnia, occurs when hyperventilation reduces the PCO2, which increases the pH  Symptoms and Signs  In acute cases (hyperventilation), there is light-headedness, anxiety, paresthesias, numbness about the mouth, and a tingling sensation in the hands and feet. Tetany occurs in more severe alkalosis from a fall in ionized calcium.  In chronic cases, findings are those of the responsible condition.  Laboratory Findings  Arterial blood pH is elevated, and PCO2 is low. Serum bicarbonate is decreased in chronic respiratory alkalosis.  Determination of appropriate compensatory changes in the HCO3– is useful to sort out the presence of an associated metabolic disorder  the changes in HCO3– values are greater if the respiratory alkalosis is chronic  Although serum HCO3– is frequently below 15 mEq/L in metabolic acidosis, it is unusual to see such a low level in respiratory alkalosis, and its presence would imply a superimposed (noncompensatory) metabolic acidosis.
  • 76. Causes of respiratory alkalosis.  Hypoxia :  Decreased inspired oxygen tension  High altitude  Ventilation/perfusion inequality  Hypotension  Severe anemia  CNS-mediated disorders  Voluntary hyperventilation  Anxiety-hyperventilation syndrome  Neurologic disease:  Cerebrovascular accident (infarction, hemorrhage)  Infection  Trauma  Tumor
  • 77. Causes…  Pharmacologic and hormonal stimulation : Salicylates, Nicotine, Xanthines ,Pregnancy (progesterone)  Hepatic failure  Gram-negative septicemia  Recovery from metabolic acidosis  Heat exposure  Pulmonary disease  Interstitial lung disease  Pneumonia  Pulmonary embolism  Pulmonary edema  Mechanical overventilation
  • 78. Treatment  Treatment is directed toward the underlying cause.  In acute hyperventilation syndrome from anxiety, reassurance , attention to underlying psychological stress and rebreathing into a paper bag will increase the PCO2.  The processes are usually self-limited since muscle weakness caused by hyperventilation-induced alkalemia will suppress ventilation.  Sedation may be necessary if the process persists however Antidepressants and sedatives should be avoided  Adrenergic blockers may ameliorate peripheral manifestations of the hyperadrenergic state.  Rapid correction of chronic respiratory alkalosis may result in metabolic acidosis as PCO2 is increased in the setting of previous compensatory decrease in HCO3–.  If respiratory alkalosis complicates ventilator management, changes in dead space, tidal volume, and frequency can minimize the hypocapnia
  • 79. Conclusion  Clinical suspicion according to scenario  Primary disorder from pH and bicarbonate or CO2 values  Degree of compensation…? Inappropriate? mixed disorder  Anion gap (corrected for serum albumin change)…? Primary metabolic acidosis  Corrected bicarbonate levels…? More than normal- associated metabolic alkalosis Less - associated metabolic non-anion gap acidosis  Review compensation for metabolic ds…confirm if initially suspected respiratory disorder exists
  • 80. Metabolic acidosis: ..see AG  High AG: …plasma osmolal gap  more.. Toxins  Less.. KA, RF, LA,  Normal AG acidosis:  urine AG….negative..GI  urinary pH ….high..RTA 1  serum potassium….high..RTA4  Metabolic alkalosis: ◦ Urinary Chloride…  Less..saline responsive…ECF contraction  more ..saline non responsive  Plasma Potassium…low..K depletion  High…Blood pressure  Plasma renin
  • 81. 1.PaCO2 equation: VCO2 x 0.863 VCO2 = CO2 production PaCO2 = ----------------- VA = VE – VD VA VE = minute (total) ventilation VD = dead space ventilation 0.863 converts units to mm Hg Condition State of PaCO2 in blood alveolar ventilation >45 mm Hg Hypercapnia Hypoventilation 35 - 45 mm Hg Eucapnia Normal ventilation <35 mm Hg Hypocapnia Hyperventilation PaCO2 reflects ratio of metabolic CO2 production to alveolar ventilation
  • 82. Hypercapnia VCO2 x 0.863 PaCO2 = ------------------ VA The only physiologic reason for elevated PaCO2 is inadequate alveolar ventilation (VA) for the amount of the body‘s CO2 production (VCO2). Since alveolar ventilation (VA) equals minute ventilation (VE) minus dead space ventilation (VD), hypercapnia can arise from insufficient VE, increased VD, or a combination.
  • 83. Hypercapnia VCO2 x 0.863 PaCO2 = ------------------ VA VA = VE – VD  Examples of inadequate VE leading to decreased VA and increased PaCO2: sedative drug overdose; respiratory muscle paralysis; central hypoventilation  Examples of increased VD leading to decreased VA and increased PaCO2: chronic obstructive pulmonary disease; severe pulmonary embolism, pulmonary edema.
  • 84. Physiologic effects of hypercapnia ◦ 1) An elevated PaCO2 will lower the PAO2 (see Alveolar gas equation), and as a result lower the PaO2. ◦ 2) An elevated PaCO2 will lower the pH (see Henderson- Hasselbalch equation). ◦ 3) The higher the baseline PaCO2, the greater it will rise for a given fall in alveolar ventilation, e.g., a 1 L/min decrease in VA will raise PaCO2 a greater amount when baseline PaCO2 is 50 mm Hg than when it is 40 mm Hg.
  • 85. Assessment of Oxygenation Status 2.Alveolar gas equation Oxygenation(alv) 3.Oxygen content equation Oxygenation(tissue)
  • 86. Oxygenation -----XXXX Diagnostics----- Parameters: /limitations Blood Gas Report 328 Pt ID 03:44 3245 / 00 Feb 5 2006 O2 Content of blood: (Hb x1.34x O2 Sat + 0.003x Dissolved O2 ) Measured 37.0 0C pH 7.452 Remember Hemoglobin pCO2 45.1 mm Hg pO2 112.3 mm Hg Corrected 38.6 0C Oxygen Saturation: pH 7.436 pCO2 47.6 mm Hg ( remember this is calculated …error prone) pO2 122.4 mm Hg Calculated Data Alveolar / arterial gradient: HCO3 act 31.2 mmol / L ( classify respiratory failure) HCO3 std 30.5 mmol / L BE 6.6 mmol / L O2 ct 15.8 mL / dl O2 Sat 98.4 % Arterial / alveolar ratio: ct CO2 32.5 mmol / L pO2 (A -a) 30.2 mm Hg  Proposed to be less variable pO2 (a/A) 0.78 Same limitations as A-a gradient Entered Data Temp 38.6 0C FiO2 30.0 % ct Hb 10.5 gm/dl
  • 87. Alveolar Gas Equation PAO2 = PIO2 - 1.2 (PaCO2) where PAO2 is the average alveolar PO2, and PIO2 is the partial pressure of inspired oxygen in the trachea PIO2 = FIO2 (PB – 47 mm Hg) FIO2 is fraction of inspired oxygen and PB is the barometric pressure. 47 mm Hg is the water vapor pressure at normal body temperature.
  • 88. Alveolar Gas Equation  If FIO2 and PB are constant, then as PaCO2 increases both PAO2 and PaO2 will decrease (hypercapnia causes hypoxemia).  If FIO2 decreases and PB and PaCO2 are constant, both PAO2 and PaO2 will decrease.  If PB decreases (e.g., with altitude), and PaCO2 and FIO2 are constant, both PAO2 and PaO2 will decrease (mountain climbing causes hypoxemia).
  • 89. P(A-a)O2  P(A-a)O2 is the alveolar-arterial difference in partial pressure of oxygen. It is commonly called the ―A-a gradient‖. It results from gravity-related blood flow changes within the lungs (normal ventilation-perfusion imbalance).  Normal P(A-a)O2 ranges from 5 to 25 mm Hg breathing room air (it increases with age). A higher than normal P(A-a)O2 means the lungs are not transferring oxygen properly from alveoli into the pulmonary capillaries. Except for right to left cardiac shunts, an elevated P(A-a)O2 signifies some sort of problem within the lungs.
  • 90. Physiologic causes of low PaO2 NON-RESPIRATORY P(A-a)O2 Cardiac right to left shunt Increased Decreased PIO2 Normal RESPIRATORY Pulmonary right to left shunt Increased Ventilation-perfusion imbalance Increased Diffusion barrier Increased Hypoventilation (increased PaCO2) Normal
  • 91. Ventilation-Perfusion imbalance  A normal amount of ventilation-perfusion (V-Q) imbalance accounts for the normal P(A-a)O2.  By far the most common cause of low PaO2 is an abnormal degree of ventilation-perfusion imbalance within the hundreds of millions of alveolar-capillary units. Virtually all lung disease lowers PaO2 via V-Q imbalance, e.g., asthma, pneumonia, atelectasis, pulmonary edema, COPD.  Diffusion barrier is seldom a major cause of low PaO2 (it can lead to a low PaO2 during exercise).
  • 92. SaO2 and oxygen content How much oxygen is in the blood? Oxygen content = CaO2 (mlO2/dl). CaO2 = quantity O2 bound + quantity O2 dissolved to hemoglobin in plasma CaO2 = (Hb x 1.34 x SaO2) + (.003 x PaO2)  Hb = hemoglobin in gm%; 1.34 = ml O2 that can be bound to each gm of Hb; SaO2 is percent saturation of hemoglobin with oxygen; .003 is solubility coefficient of oxygen in plasma: .003 ml dissolved O2/mm Hg PO2.
  • 93. Given arterial oxygen saturation (SpO2) = 100%, Hb = 15 g/100 ml and arterial partial pressure of oxygen (PaO2) = 13.3 kPa, then the oxygen content of arterial blood (CaO2) is: CaO2 = 20.1 +0.3 = 20.4 ml/100 ml  Similarly the oxygen content of mixed venous blood can be calculated. Given normal values of mixed venous oxygen saturation (SvO2) = 75% and venous partial pressure of oxygen (PvO2) = 6 kPa, so: CvO2 = 15.2 + 0.1 = 15.2 ml/100 ml
  • 94. Oxygen dissociation curve: SaO2 vs. PaO2 Also shown are CaO2 vs. PaO2 for two different hemoglobin contents: 15 gm% and 10 gm%. CaO2 units are ml O2/dl. P50 is the PaO2 at which SaO2 is 50%.
  • 95. SaO2 – is it calculated or measured? SaO2 is measured in a ‗co-oximeter‘. The traditional ‗blood gas machine‘ measures only pH, PaCO2 and PaO2,, whereas the co- oximeter measures SaO2, carboxyhemoglobin, methemoglobin and hemoglobin content. Newer ‗blood gas‘ consoles incorporate a co- oximeter, and so offer the latter group of measurements as well as pH, PaCO2 and PaO2. Always make sure the SaO2 is measured, not calculated. If it is calculated from the PaO2 and the O2-dissociation curve, it provides no new information, and could be inaccurate -- especially in states of CO intoxication or excess methemoglobin. CO and metHb do not affect PaO2, but do lower the SaO2.
  • 96. Carbon monoxide – an important cause of hypoxemia  Normal %COHb in the blood is 1-2%, from metabolism and small amount of ambient CO (higher in traffic-congested areas)  All smokers have excess CO in their blood.  CO binds @ 200x more avidly to hemoglobin than O2, displacing O2 from the heme binding sites.  CO : 1) decreases SaO2 by the amount of %COHb present, and 2) shifts the O2-dissociation curve to the left, retarding unloading of oxygen to the tissues.  CO does not affect PaO2, only SaO2. To detect CO poisoning, SaO2 and/or COHb must be measured (requires co- oximeter). In the presence of excess CO, SaO2 (when measured) will be lower than expected from the PaO2.
  • 97. CO does not affect PaO2! A patient presented to the ER with headache and dyspnea.  His first blood gases showed PaO2 80 mm Hg, PaCO2 38 mm Hg, pH 7.43. SaO2 on this first set was calculated from the O2-dissociation curve at 97%, and oxygenation was judged normal.  He was sent out from the ER and returned a few hours later with mental confusion; this time both SaO2 and COHb were measured (SaO2 shown by ‗X‘): PaO2 79 mm Hg, PaCO2 31 mm Hg, pH 7.36, SaO2 53%, carboxyhemoglobin 46%. CO poisoning was missed on the first set of blood gases because SaO2 was not measured!
  • 98. Causes of Hypoxia 1. Hypoxemia (=low PaO2 and/or low CaO2) ◦ a. reduced PaO2 – usually from lung disease (most common physiologic mechanism: V-Q imbalance) ◦ b. reduced SaO2 -- most commonly from reduced PaO2; other causes include carbon monoxide poisoning, methemoglobinemia, or rightward shift of the O2- dissociation curve ◦ c. reduced hemoglobin content -- anemia 2. Reduced oxygen delivery to the tissues ◦ a. reduced cardiac output -- shock, congestive heart failure ◦ b. left to right systemic shunt (as may be seen in septic shock) 3. Decreased tissue oxygen uptake ◦ a. mitochondrial poisoning (e.g., cyanide poisoning) ◦ b. left-shifted hemoglobin dissociation curve (e.g., from acute alkalosis, excess CO, or abnormal hemoglobin structure)
  • 99. Arterial Oxygen Tension (PaO2)  Normal value in healthy adult breathing room air at sea level 97 mm Hg.  progressively with age  Dependant upon 1. FiO2 2. Patm  Hypoxemia is PaO2 < 80 mm Hg at RA  Most pts who need ABG usually req O2 therapy  O2 therapy should not be withheld/interrupted ‗to determine PaO2 on RA‘
  • 100. Acceptable PaO2 Values on Room Air Age Group Accepable PaO2 (mm Hg) Adults upto 60 yrs > 80 & Children Newborn 40-70 70 yrs > 70 80 yrs > 60 90 yrs > 50 60 yrs 80 mm Hg  1mm Hg/yr
  • 101. Inspired O2 – PaO2 Relationship FIO2 (%) Predicted Min PaO2 (mm Hg) 30 150 40 200 50 250 80 400 100 500 If PaO2 < FIO2 x 5, pt probably hypoxemic at RA
  • 102. Hypoxemia on O2 therapy  Uncorrected: PaO2 < 80 mm Hg (< expected on RA & FIO2)  Corrected: PaO2 = 80-100 mm Hg (= expected on RA but < expected for FIO2)  Excessively Corrected: PaO2 > 100 mm Hg (> expected on RA but < expected for FIO2)  PaO2 > expected for FIO2: 1. Error in sample/analyzer 2. Pt‘s O2 consumption reduced 3. Pt does not req O2 therapy (if 1 & 2 NA)
  • 103. Thanks